SlideShare a Scribd company logo
1 of 117
IAS Prelims 2017 Analysis
Civilsdaily.com
Overview
Civilsdaily has been trying to gauge trends followed by USPC in recent years by
applying various Data interpretation tools.
This is one such attempt to delve more into UPSC analysis and get some insights
about various factors involved in Prelims papers.
Understanding the Trend
Subject 2011 2012 2013 2014 2015 2016 2017
History 11 19 16 20 17 15 13
Geography 11 17 18 14 16 7 10
Polity 12 20 16 14 13 7 26
Economy 19 17 19 10 13 18 8
E & B 15 17 17 18 11 18 10
Science 19 9 14 16 8 8 10
CA + GK 13 1 0 8 22 27 23
How have
divisions been
done?
The divisions are
All subjects bifurcated according to
their static and dynamic
Subject wise analysis- Division into
sub-subjects, difficulty levels,
static and dynamic divisions and
others.
Trend analysis
Findings
Current Affairs have been increasing year on
year and even static questions have effect of
CA on them.
Implications:
More focus on CA has to made
Comprehensive plan is needed where CA
go along with static portion and not
separately.
Static
Dynamic (CA based)
2014
92
8
2015
78
22
2016
73
27
2017
78
22
Trend analysis- 2017
Trend analysis- 2017
Trend analysis- 2017
Trend analysis- 2017
Part 1 - Economy
High and Medium level
Questions
Economics Sub Topics Count of Question Static/Dynamic Difficulty Level
Banking, Finance 1 Static Medium
Budget, Fiscal Policy 1 Static Medium
Important Intl. Organizations 2 Static Medium
Infrastructure 1 Static Medium
Organizations involved 2 Static Medium
Sectors of Economy 1 Static High
Grand Total 8
Level : High
Q.1 Consider the following statements:
1. Tax revenue as a percent of GDP of India has steadily increased
in the last decade.
2. Fiscal deficit as a percent of GDP of India has steadily increased in
the last decade.
Which of the statements given- above is/are correct ?
(a) 1 only
(b) 2 only
(c) Both 1 and 2
(d) Neither 1 nor 2
Answer- a
Standard Explanation
Fiscal Deficit has
decreased in the last
decade.
www.civilsdaily.com/blog/
discussing-budget-2016-17-
fiscal-discipline/
IAS Ranker's Inputs
Read Economic survey
and make short notes to
tackle such questions.
K Siddhartha Sir's Inputs
Difficult if you haven’t
followed the Eco Survey
over time.
Needs your ability to
remember growth trends
for atleast 2+ years
Level : Medium
Q.2 Consider the following statements:
1. The Standard Mark of Bureau of Indian Standards (BIS) is mandatory for
automotive tyres and tubes.
2. AGMARK is a quality Certification Mark issued by the Food and Agriculture
Organisation (FAO).
Which of the statements given above is/are correct?
(a) 1 only
(b) 2only
(c) Both 1 and 2
(d) Neither 1 nor 2
Answer- a
Standard Explanation
Under Product Certification ,
Mandatory certification is there
for Automobile Accessories
such as Automotive vehicles –
Tubes and pneumatic tyres
AGMARK (Agricultural Mark)
is a certification issued by
Directorate of Marketing and
Inspection for agricultural
produce.
IAS Ranker's Inputs
There are some popular
certifications such as
AGMARK, GRIHA rating,
gold's Hallmark etc. One
should know basics about
them. Also, BIS is important
Indian agency. One should
have idea about its
functioning, its ministry etc.
K Siddhartha Sir's Inputs
AGMARK is by India which
eliminates option (b) and (c).
Level : Medium
Q.3 With reference to the ‘National Intellectual Property Rights Policy’, consider the
following statements:
1. It reiterates India’s commitment to the Doha Development Agenda and the
TRIPS Agreement.
2. Department of Industrial Policy and Promotion is the nodal agency for regulating
intellectual property rights in India.
Which of the above statements is/are correct?
(a) 1 only
(b) 2 only
(c) Both 1 and 2
(d) Neither 1 nor 2
Answer- c
Standard Explanation
"The National IPR Policy recognizes
that India has a well-established TRIPS-
compliant legislative, administrative
and judicial framework to safeguard
IPRs. It reiterates India’s commitment
to the Doha Development Agenda and
the TRIPS agreement.
It is monitored by DIPP (the nodal
department to coordinate, guide and
oversee implementation and future
development of IPRs in India)
www.civilsdaily.com/story/intellectual-
property-rights-regime/
IAS Ranker's Inputs
For prelims, one must know
following points about
important government
policies - nodal agencies,
targets, related global
agreements/conventions
.
K Siddhartha Sir's Inputs
DIPP monitors IPR issues
(Remember in context of GI
items etc.)
Level : Medium
Q.4 What is the purpose of setting up of Small Finance Banks (SFBs) in India?
1. To supply credit to small business units
2. To supply credit to small and marginal farmers
3. To encourage young entrepreneurs to set up business particularly in rural areas.
Select the correct answer using the code given below:
(a) 1 and 2 only
(b) 2 and 3 only
(c) 1 and 3 only
(d) 1, 2 and 3
Answer- a
Standard Explanation
"Local focus and ability to serve
smaller customers will be a key
criterion in licensing such banks.
The bank shall primarily
undertake basic banking
activities of accepting deposits
and lending to small farmers,
small businesses, micro and
small industries, and
unorganized sector entities.
www.civilsdaily.com/small-
finance-banks-background/
IAS Ranker's Inputs
For prelims, one must know
recent schemes in
details - their purpose,
beneficiary, nodal
agency, funds (centre :
state)
K Siddhartha Sir's Inputs
In such options format, prove
any one of the statements
wrong. Here, statement 3 is
wrong which leaves only
option (a).
Level : Medium
Q.5 Which of the following statements is/are correct regarding the Monetary Policy
Committee (MPC)?
1. It decides the RBI’s benchmark interest rates.
2. It is a 12-member body including the Governor of RBI and is reconstituted every
year.
3. It functions under the chairmanship of the Union Finance Minister.
Select the correct answer using the code given below:
(a) 1 only
(b) 1 and 2 only
(c) 3 only
(d) 2 and 3 only
Answer- a
Standard Explanation
The Monetary Policy Committee
would be entrusted with the task
of fixing the benchmark policy
rate (repo rate) required to
contain inflation within the
specified target level.
It consists of 6 members.
The Governor of the Bank—
Chairperson, ex officio
www.civilsdaily.com/centre-
notifies-amended-rbi-act-mpc/
IAS Ranker's Inputs
Whenever new institution is
created by government,
one must know the
purpose of creation, its
structure, and ministry
under which it
functions, mode of
creation (statutory,
executive,
constitutional) etc.
K Siddhartha Sir's Inputs
(EIBQ).
Level : Medium
Q.6 Consider the following statements:
1. India has ratified the Trade Facilitation Agreement (TFA) of WTO.
2. TFA is a part of WTO’s Bali Ministerial Package of 2013.
3. TFA came into force in January 2016.
Which of the statements given above is/are correct?
a) 1 and 2 only
b) 1 and 3 only
c) 2 and 3 only
d) 1, 2 and 3
Answer- a
Standard Explanation
WTO members concluded
negotiations at the 2013 Bali
Ministerial Conference on the
landmark Trade Facilitation
Agreement (TFA), which entered
into force on 22 February 2017
following its ratification by two-
thirds of the WTO membership.
www.civilsdaily.com/story/wto-
and-india/
IAS Ranker's Inputs
TFA is important global
agreement. For prelims, one
must know the timeline and
India's efforts to implement
the agreement.
K Siddhartha Sir's Inputs
WTO works on the principle
of consensus. India has ratified
the TFA in mid 2016. So, it
cannot come into force in Jan
2016. By that logic, Statement
3 is wrong and that leaves only
option a as correct answer.
Level : Medium
Q.7 With reference to ‘National Investment and Infrastructure Fund’, which of the
following statements is/are correct?
1. It is an organ of NITI Aayog.
2. It has a corpus of Rs. 4,00,000 crore at present.
Select the correct answer using the code given below :
(a) 1 only
(b) 2 only
(c) Both 1 and 2
(d) Neither 1 nor 2
Answer- d
Standard Explanation
NIIF has been structured as a
fund of funds and set up as
Category II Alternate Investment
Fund (AIF) under the Securities
and Exchange Board of India
(SEBI) Regulations.
Total corpus of the fund is Rs.
40000 Crore.
www.civilsdaily.com/learn-about-
niif/
IAS Ranker's Inputs
Make notes of important
initiatives, their purpose,
implementing agency, fund
allocation etc.
K Siddhartha Sir's Inputs
(EIBQ)
Further, 4 lakh crore seems
very high figure. So, option d
can be guessed by that logic.
Level : Medium
Q.8 "The Global Infrastructure Facility is a/an
(a) ASEAN initiative to upgrade infrastructure in Asia and financed by credit from
the Asian Development Bank.
(b) World Bank collaboration that facilitates the preparation and structuring of
complex infrastructure Public-Private Partnerships (PPPs) to enable mobilization of
private sector and institutional investor capital.
(c) Collaboration among the major banks of the world working with the OECD and
focused on expanding the set of infrastructure projects that have the potential to
mobilize private investment.
(d) UNCTAD funded initiative that seeks to finance and facilitate infrastructure
development in the world."
Answer- b
Standard Explanation
The World Bank (WB) has
launched the GIF to specifically
cater to the infrastructure
needs of the emerging
economies and developing
countries.
The GIF will channel money
towards bankable
infrastructure project in such
countries.
IAS Ranker's Inputs
Make notes of important
initiatives, their purpose,
implementing agency, fund
allocation etc.
K Siddhartha Sir's Inputs
Very close and overlapping
choices makes it tough
reading and choosing with an
equal pinch for knowledge
and information. Difficult to
answer.
Part 2 - Polity
High and Medium level
Questions
Polity Sub Topics No. of questions
Amendments to constitution 1
Elections- LS, RS, States 3
Functions of Judiciary 1
Functions of Legislature 5
Fundamental Rights/DPSP 7
Govt. Schemes 7
Important articles/schedules 2
Important treaties/ protocols 1
Misc. 1
Organizations involved 1
Panchayati Raj- Local governance 1
Powers of different authorities 2
Grand Total 32
Level : High
Q.1 Who among the following can join the National Pension System
(NPS)?
(a) Resident Indian citizens only
(b) Persons of age from 21 to 55 only
(c) All State Government employees joining the services after the
date of notification by the respective State Governments
(d) All Central Governments Employees including those of Armed
Forces joining the services on or after 1st April, 2004"
Answer- c
Standard Explanation
"The Central Government had
introduced the National Pension
System (NPS) with effect from
January 1, 2004 (except for armed
forces).
Subsequently, various State
Governments adopted this
architecture and implemented NPS
with effect from different dates.
www.civilsdaily.com/learn-about-
national-pension-system/
IAS Ranker's Inputs
Create a list of important
schemes where basic details
like beneficiary, nodal
ministry, targets etc.
Use CD’s Prelims Listicles for
quick revision:
http://www.civilsdaily.com/top
ic/pre-compilations-govt-
schemes-2016-17-in-quick-
reference-excel-sheet/
K Siddhartha Sir's Inputs
Pension has been a problem.
Candidate has been asked the
question to know of he/she
showed enough concerns. His
concerns and sensitivity about
the problems of the society. His
empathy would determine his
observation and answer to this
topics.
Level : High
Q.2 With reference to `Quality Council of India (QCI)’, consider the
following
Statements:
1. QCI was set up jointly by the Government of India and the Indian
Industry.
2. Chairman of QCI is appointed by the Prime Minister on the
recommendations of the industry to the Government.
Which of the above statements is/are correct?
(a) 1 only
(b) 2 only
(c) Both 1 and 2
Answer- c
Standard Explanation
Formed in 1997, the Quality
Council of India was set-up by the
CII, the FICCI, and the Associated
Chambers of Commerce and
Industry.
The Chairman of QCI is
appointed by the Prime Minister
on recommendation of the
industry to the government.
IAS Ranker's Inputs
Read 'About Us' webpage
from official website and
make their short notes for
prelims. Also, carefully
observe in newspaper when a
new appointment is made to
any government agency. The
QCI was in news related to
Swachh Bharat Mission.
K Siddhartha Sir's Inputs
Pure, blue bloodied fact based
question to know whether the
student keeps abreast of certain
things……or not.
Level : High
Q.3 ‘Democracy’s superior virtue lies in the fact that it calls into
activity
(a) the intelligence and character ordinary men and women.
(b) the methods for strengthening executive leadership
(c) a superior individual with dynamism and vision.
(d) a band of dedicated party workers.
Answer- a
Standard Explanation
A democracy is superior,
because it allows a right to every
adult citizen, without any
educational and wealth criterion.
This allows him to use his own
intelligence in choosing
representatives.
IAS Ranker's Inputs
The question required clarity
in fundamentals of polity and
careful and patient reading of
options.
To make clarity in polity, read
some good polity books like
NCERT, Subhash Kashyap.
K Siddhartha Sir's Inputs
This is almost funny.
It is an essay type topic.
Requires a deep thinking to
answer. Fortunately the choices
are not close. So, you could have
used Tikdams to answer
Such topics have made their
entry into PT questions this
time.
Level : High
Q.4 In the context of India, which one of the following is the correct
relationship
between Rights and Duties?
(a) Rights are correlative with Duties.
(b) Rights are personal and hence independent of society and
Duties.
(c) Rights, not Duties, are important for the advancement of the
personality of the citizen.
(d) Duties, not Rights, are important for the stability of the State.
Answer- a
Standard Explanation
Rights and duties are two phases
of the same thing. Rights are
considered to be essential for the
expansion of human personality.
Rights can be enjoyed only in the
world of duties. For every right
there is corresponding duty.
IAS Ranker's Inputs
Fundamentals of Polity must
be understood.
This requires in-depth
understanding
K Siddhartha Sir's Inputs
Understand this language.
Unless the candidates decipher
the language, it will be even
more difficult now onwards to
answer such questions.
Level : High
Q.5 With reference to the ‘Prohibition of Benami Property Transactions Act,
1988 (PBPT Act)’, consider the following statements:
1. A property transaction is not treated as a benami transaction if the owner of
the property
is not aware of the transaction.
2. Properties held benami are liable for confiscation by the Government.
3. The Act provides for three authorities for investigations but does not provide
for any
appellate mechanism.
Which of the statements given above is/are correct?
a) 1 only
b) 2 only
c) 1 and 3 only
Answer- b
Standard Explanation
"Benami Transactions (Prohibition)
Act, 1988 prohibits certain types of
financial transactions. The act defines a
‘benami’ transaction as any transaction
in which property is transferred to one
person for a consideration paid by
another person. The act bans all
benami transactions and gives the
government the right to recover
property held benami without paying
any compensation.
www.civilsdaily.com/new-benami-law-
to-come-into-effect-from-november-1/
IAS Ranker's Inputs
The Act was recently
amended and was major
instrument in government's
battle against black money.
Use CD’s Prelims Listicles for
quick revision:
http://www.civilsdaily.com/top
ic/pre-compilations-govt-
schemes-2016-17-in-quick-
reference-excel-sheet/
K Siddhartha Sir's Inputs
Good analytical frame of mind
and learning required for
processing this information.
NO easy choices. No easy
options.
Level : Medium
Q.6 With reference to the Parliament of India, consider the following
statements:
1. A private member’s bill is a bill presented by a Member of
Parliament who is not elected but only nominated by the President
of India.
2. Recently, a private member’s bill has been passed in the
Parliament of India for the first time in its history.
Which of the statements given above is/are correct?
(a) 1 only
(b) 2 only
(c) Both 1 and 2
Answer- d
Standard Explanation
Members of parliament other than ministers
are called private members and bills presented
by them are known as private member’s bills.
Any MP can introduce a bill in parliament.
Only 14 private members’ bills passed since
Independence. The Rights of Transgender
Persons Bill, 2014, is the first private member’s
bill to get the upper house’s approval in the
past 45 years.
www.civilsdaily.com/topic/transgender-bill-
key-issues-and-analysis/
IAS Ranker's Inputs
The private member's
bill was in news recently
when Rajya Sabha
passed a bill for
Transgender
Community.
That made the basics of
Private bills important
for prelims.
K Siddhartha Sir's Inputs
Simple, straight and Basics.
Need to study.
Q.7 ‘Recognition of Prior Learning Scheme’ is sometimes
mentioned in the news
with reference to
(a) Certifying the skills acquired by construction workers through
traditional channels.
(b) Enrolling the persons in Universities for distance learning
programmes.
(c) Reserving some skilled jobs to rural and urban poor in some
public sector undertakings.
(d) Certifying the skills acquired by trainees under the National Skill
Development
Answer- a
Standard Explanation
"Recognition of Prior Learning
Scheme- Individuals with prior
learning experience or skills shall be
assessed and certified under the
Recognition of Prior Learning (RPL)
component of the Pradhan Mantri
Kaushal Vikas Yojana (PMKVY).
RPL aims to align the competencies
of the unregulated workforce of the
country to the NSQF (National Skills
Qualification Framework)
IAS Ranker's Inputs
Here option A is correct. It is
not about certifying trainees
but those who already possess
skills (ie prior learning).
In addition, the scheme is
launched for construction
sector.
K Siddhartha Sir's Inputs
Previously, these questions
would have been asked in match
the following type. This has
changed.
Meaning pure factual content
has given way to some degree of
inspection, introspection and
retrospection.
Level : Medium
Q.8 Local self-government can be best explained as an exercise in
a) Federalism
b) Democratic decentralization
c) Administrative delegation
d) Direct democracy
Answer- b
Standard Explanation
Democratic decentralization
means decentralization of
power.
The main purpose of democratic
decentralization is to bring
fundamental changes in the
traditional outlook about the
power structure of the
government.
IAS Ranker's Inputs
Read and understand basics of
Panchayati Raj System.
For prelims, the evolution of PRI,
the provisions of 73rd and 74th
amendments, PESA 1996 must be
known to aspirant.
In addition to that, administrative
machinery in 5th schedule and 6th
schedule area must also be known.
K Siddhartha Sir's Inputs
The question is the Basis of
Polity but with close
observation. (EIBQ)
Level : Medium
Q.9 Consider the following statements:
With reference to the Constitution of India, the Directive Principles of State Policy
constitute limitations upon
1. legislative function
2. executive function
Which of the above statements is/are correct?
a) 1 only
b) 2 only
c) Both 1 and 2
d) Neither 1 nor 2
Answer- d
Standard Explanation
DPSP does not place any
limitation on legislative and
executive function, it is simply
a guideline.
IAS Ranker's Inputs
One cannot go judiciary for
violation of DPSP by
legislature or executive if they
are not provided by any law.
K Siddhartha Sir's Inputs
The question is limitations of
Directive Policy-- that means
studying the topic closely,
minutely. No hard work but
very minute work.
No memorization but a study in
a calm atmosphere to avoid
cluttering of mind.
Q.10 Which of the following are the objectives of ‘National Nutrition
Mission’?
1. To create awareness relating to malnutrition among pregnant women
and lactating mothers.
2. To reduce the incidence of anaemia among young children, adolescent
girls and women.
3. To promote the consumption of millets, coarse cereals and unpolished
rice.
4. To promote the consumption of poultry eggs.
Select the correct answer using the code given below:
a) 1 and 2 only
b) 1, 2 and 3 only
c) 1, 2 and 4 only
Answer- a
Standard Explanation
The key objectives of this programme
are :
To create awareness relating to
malnutrition amongst pregnant
women, lactating mothers, promote
healthy lactating practices and
importance of balanced nutrition;
To reduce incidence of anaemia among
young children, adolescent girls and
women.
www.civilsdaily.com/op-ed-snap-need-
nutrition-mission/
IAS Ranker's Inputs
Make ministry wise short
notes on schemes and
missions. Read their
objectives, beneficiaries,
targets and implementing
agencies.
Make use of the last minute
Prelims listicles
K Siddhartha Sir's Inputs
Straight simple, typical,
representative question from
Govt. Schemes.
Level : Medium
Q.11 Consider the following statements:
1. In the election for Lok Sabha or State Assembly, the winning candidate
must get at least 50 percent of the votes polled, to be declared elected.
2. According to the provisions laid down in the Constitution of India, in
Lok Sabha, the Speaker’s post goes to the majority party and the
Deputy Speaker’s to the Opposition.
Which of the statements given above is/are correct?
a) 1 only
b) 2 only
c) Both 1 and 2
d) Neither 1 nor 2
Answer- d
Standard Explanation
India follows First Past the Post
system, there is no provision
for 50% votes. There is no
provision related to it in the
Constitution.
IAS Ranker's Inputs
For prelims, one must be
aware of the some
Parliamentary practices
whether they are based on
convention, law or
constitutional provision.
Other examples include Zero
hour, Deputy Chairman of
Rajya Sabha etc.
K Siddhartha Sir's Inputs
In the first option, Provision of
50% post is essentially a ploy to
confuse the candidates.
The purpose of the question is
to make sure whether the
students are 100% sure about
the answers or not.
Level : Medium
Q.12 What is the purpose of Vidyanjali Yojana’?
1. To enable the famous foreign campuses in India.
2. To increase the quality of education provided in government schools by taking help
from the private sector and the community.
3. To encourage voluntary monetary contributions from private individuals and
organizations so as to improve the infrastructure facilities for primary and secondary
schools.
Select the correct answer using the code given below:
a) 2 only
b) 3 only
c) 1 and 2 only
d) 2 and 3 only
Answer- a
Standard Explanation
Vidyanjali is an initiative to
enhance community and
private sector involvement in
Government run elementary
schools across the country
under the overall aegis of the
Sarva Shiksha Abhiyan.
www.civilsdaily.com/app-aid-
volunteer-workers-govt-
schools/
IAS Ranker's Inputs
For prelims, one must know
recent schemes in details -
their purpose, beneficiary,
nodal agency, funds (centre :
state)
K Siddhartha Sir's Inputs
The question is Ok but the
options are very close, very
interrelated, very meaningful.
Such genre of questions were
very few last year but are more
in number this year.
You have to be thorough with
your reading an revision.
Level : Medium
Q.13 Out of the following statements, choose the one that brings out the principle
underlying the Cabinet form of Government :
(a) An arrangement for minimizing the criticism against the Government whose
responsibilities are complex and hard to carry out to the satisfaction of all.
(b) A mechanism for speeding up the activities of the Government whose
responsibilities are increasing day by day.
(c) A mechanism of parliamentary democracy for ensuring collective responsibility of
the Government to the people.
(d) A device for strengthening the hands of the head of the Government whose hold
over the people is in a state of decline.
Answer- c
Standard Explanation
Council of Ministers is
collectively responsible to
Parliament (Lok Sabha) and
Lok Sabha is elected by people
themselves.
IAS Ranker's Inputs
Fundamental question from
Polity.
K Siddhartha Sir's Inputs
Although this question was
simple, Simple to ask, simple to
answer and simple to think
there were overtones to make it
complicated.
Level : Medium
Q.14 With reference to ‘National Skills Qualification Framework (NSQF)’, which of the
statements given below is/are correct?
1. Under NSQF, a learner can acquire the certification for competency only through
formal learning.
2. An outcome expected from the implementation of NSQF is the mobility between
vocational and general education.
Select the correct answer using the code given below:
(a) 1 only
(b) 2 only
(c) Both 1 and 2
(d) Neither 1 nor 2
Answer- b
Standard Explanation
"The National Skills Qualifications
Framework (NSQF) is a
competency-based framework that
organizes all qualifications according
to a series of levels of knowledge,
skills and aptitude. These levels, are
defined in terms of learning
outcomes which the learner must
possess regardless of whether they
are obtained through formal, non-
formal or informal learning.
IAS Ranker's Inputs
Prepare notes on skill
development in India.
For prelims, it should include
the current status of skills in
India, current programmes
run by government to
improve skills like Skill India,
the agencies involved in skill
development in India.
K Siddhartha Sir's Inputs
Contemporary,
Current
Compact with deep information
levels and differentiation skill,
i.e. ability to differentiate
between different type of
choices.
Level : Medium
Q.15 Consider the following in respect of ‘National Career Service’ :
1. National Career Service is an initiative of the Department of Personnel
and Training, Government of India.
2. National Career Service has been launched in a Mission Mode to improve
the employment opportunities to uneducated youth of the country.
Which of the above statements is/are correct ?
(a) 1 only
(b) 2 only
(c) Both 1 and 2
(d) Neither 1 nor 2
Answer- b
Standard Explanation
The Government is implementing
the National Career Service
Project for linking employment
exchanges and other institutions
using technology to provide a
variety of employment related
services like job postings, career
counselling, vocational guidance,
skill courses, apprenticeship, etc.
www.civilsdaily.com/pib-
implementation-of-national-
career-service/
IAS Ranker's Inputs
Important scheme related to
employment.
K Siddhartha Sir's Inputs
The candidate need Concerns of
unemployment, which is
frequently covered in
Newspaper as it also concerns
the youth.
Part 3 - Geography
High and Medium level
Questions
Geography Sub Topics No. of Questions
Agriculture- Irrigation, fertilizers etc. 1
Indian Geography-Forests 1
Indian Geography-Places 3
Indian Geography-Rivers 1
World Geography 4
Grand Total 10
Level : High
Q.1 From the ecological point of view, which one of the following assumes
importance in being a good link between the Eastern Ghats and the Western
Ghats?
(a) Sathyamangalam Tiger Reserve
(b) Nallamala Forest
(c) Nagarhole National Park
(d) Seshachalam Biosphere Reserve
Answer- b
Standard Explanation
Sathyamangalam forest range is a
significant wildlife corridor in
the Nilgiri Biosphere Reserve
between the Western Ghats and
the rest of the Eastern Ghats.
It is located in Tamil Nadu.
http://www.civilsdaily.com/topic/
15-nov-2016-prelims-daily-with-
previous-year-questions-tikdams/
IAS Ranker's Inputs
Mark important protected
areas on blank maps. Along
with that, know about
important species in those
areas. Revise them regularly.
K Siddhartha Sir's Inputs
Answers to these type of questions
now requires identifying the most
significant part, most unique part, i.e.,
with respect to its location, river,
animals, flora, or any new
development.
For example asking something like
northern most confluence town in
India?
Level : High
Q.2 At one of the places in India, if you stand on the seashore and watch
the sea, you will find that the sea water recedes from the shore line a few
kilometers and comes back to the shore, twice a day, and you can actually
walk on the seafloor when the water recedes. This unique phenomenon is
seen at:
a) Bhavnagar
b) Bheemunipatnam
c) Chandipur
d) Nagapattinam
Answer- c
Standard Explanation
The beach is unique in that the
water recedes up to 5
kilometers during the ebb tide.
When the tide is in, locals trawl
for small fish along the coast.
IAS Ranker's Inputs
Very factual question.
K Siddhartha Sir's Inputs
No theory, no memorisation, no
fact, but one unique information
separated this beach out of context.
This beach like Dhanu road is
remarkable for a lot of things.
Identification of uniqueness is one
key factor that is emerging now.
Level : High
Q.3 Consider the following statements:
1. In India, the Himalayas are spread over five States only.
2. Western Ghats are spread over five States only.
3. Pulicat Lake is spread over two States only.
Which of the statements given above is/are correct ?
(a) 1 and 2 only
(b) 3 only
(c) 2 and 3 only
(d) 1 and 3 only
Answer- b
Standard Explanation
Western Ghats – The Ghats
traverse the States of Kerala,
Tamil Nadu, Karnataka, Goa,
Maharashtra and Gujarat.
Pulicat Lake is the second
largest brackish water lake or
lagoon in India, after Chilka
Lake. It straddles the border of
Andhra Pradesh and Tamil
Nadu states.
IAS Ranker's Inputs
Factual question/map needs to
be fully memorized in order to
answer.
K Siddhartha Sir's Inputs
Observation based question,
location based question, one
that cannot be memorised.
Level : Medium
Q.4 Mediterranean Sea is a border of which of the following countries ?
1. Jordan
2. Iraq
3. Lebanon
4. Syria
Select the correct answer using the code given below:
(a) 1, 2 and 3 only
(b) 2 and 3 only
(c) 3 and 4 only
(d) 1, 3 and 4 only
Answer- c
Standard Explanation
The countries with coastlines on
the Mediterranean Sea are
Albania, Algeria, Bosnia and
Herzegovina, Croatia, Cyprus,
Egypt, France, Greece, Israel,
Italy, Lebanon, Libya, Malta,
Morocco, Monaco, Montenegro,
Slovenia, Spain, Syria, Tunisia
and Turkey.
IAS Ranker's Inputs
For prelims, carefully study
the middle east, south east
areas.
K Siddhartha Sir's Inputs
(EIBQ)
Level : Medium
Q.5 With reference to ‘Indian Ocean Dipole (IOD)’ sometimes mentioned in the
news while forecasting Indian monsoon, which of the following statements is/are
correct?
1. IOD phenomenon is characterized by a difference in sea surface temperature
between tropical Western Indian Ocean and tropical Eastern Pacific Ocean.
2. An IOD phenomenon can influence an El Nino’s impact on the monsoon.
Select the correct answer using the code given below:
(a) 1 only
(b) 2 only
(c) Both 1 and 2
(d) Neither 1 nor 2
Answer- b
Standard Explanation
The phenomenon is concerned
to temperature differentials in
Western Indian Ocean and
Eastern Indian Ocean.
www.civilsdaily.com/story/geog
raphic-phenomenon-key-
concepts-and-issues/
IAS Ranker's Inputs
The term was in news last year
for its impact on monsoon.
Make notes of such important
terms appearing in news.
K Siddhartha Sir's Inputs
Imagine the question, It’s not on
El Nino, Not On ENSO, Not on
Madden Julien oscillation, not on
Aido Arabian Sea temperature
inversion.
Of all the factors that may have
affected the monsoons the pic
was dipole. Of course all them
are linked by ENSO event.
Level : Medium
Q.6 Which of the following practices can help in water conservation in agriculture ?
1. Reduced or zero tillage of the land
2. Applying gypsum before irrigating the field
3. Allowing crop residue to remain in the field
Select the correct answer using the code given below :
(a) 1 and 2 only
(b) 3 only
(c) 1 and 3 only
(d) 1, 2 and 3
Answer- d
Standard Explanation
All these methods help in
reducing water usage..
IAS Ranker's Inputs
Agriculture consumes a major
portion of water and due to
scarcity of fresh water, its
conservation techniques in
agriculture become very
important. For prelims, one
must know about Climate
smart agriculture, organic
farming, sustainable
agriculture etc.
K Siddhartha Sir's Inputs
A lot of discussion is carried
on about saving water and the
correct method of agriculture.
So the students must
understand the importance of
water conservation
particularly in agriculture
where a lot of water is wasted.
Part 4 - Environment
& Biodiversity
High and Medium level
Questions
E&B Sub Topics No. of questions
Agriculture- Irrigation, fertilizers etc. 1
Basic terms related to E&B 1
Important articles/schedules 2
Important Intl. Organizations 1
Important treaties/ protocols 2
Indian Geography-Places 2
New Tech/ Innovation in news 1
Organizations involved 1
Static theory on E&B 1
Grand Total 12
Level : High
Q.1 Consider the following statements :
1. Climate and Clean Air Coalition (CCAC) to Reduce Short Lived Climate
Pollutants is a unique initiative of G20 group of countries;
2. The CCAC focuses on methane, black carbon and hydrofluorocarbons.
Which of the statements given above is/are correct ?
(a) 1 only
(b) 2 only
(c) Both 1 and 2
(d) Neither 1 nor 2
Answer- b
Standard Explanation
The Climate and Clean Air
Coalition to Reduce Short-Lived
Climate Pollutants (CCAC) was
launched by the United Nations
Environment Programme
(UNEP) and six countries—
Bangladesh, Canada, Ghana,
Mexico, Sweden, and the United
States—on 16 February 2012. It
focuses on O3, CH4, Black
Carbon, and HFCs.
IAS Ranker's Inputs
Since it is 4 - 5 years old news
and India is not party to this
coalition, It becomes very
difficult to know about such
question.
In prelims, one must also
know which questions to leave
because there are negative
marks too. One should not
make total blind guesses.
K Siddhartha Sir's Inputs
The explanation says all.
Of course, All ecological conventions
taking place must be considered
important.
The back ground history for
conventions need to be added. No
more cosmetic preparation required.
Level : High
Q.1 With, reference to ‘Global Climate Change Alliance’ which of the following
statements is/are correct?
1. It is an initiative of the European Union.
2. It provides technical and financial support to targeted developing countries to
integrate climate change into their development policies and budgets.
3. It is coordinated by World Resources Institute (WRI) and World Business Council
for Sustainable Development (WBCSD).
Select the correct answer using the code given below :
(a) 1 and 2 only
(b) 3 only
(c) 2 and 3 only
(d) 1, 2 and 3
Answer- a
Standard Explanation
"The Global Climate Change
Alliance (GCCA) is an initiative
of the European Union. Its
overall objective is to build a new
alliance on climate change
between the European Union
and the poor developing
countries that are most affected
and that have the least capacity
to deal with climate change.
IAS Ranker's Inputs
Try getting answer from
analysing options.
K Siddhartha Sir's Inputs
This question will make the student
think for sometime before they can
gather their nerve to answer it
correctly, what a way to tease the
students by putting option 3 and its
associated institutions.
Expect more from UPSC next year
Level : High
Q.3 Recently there was a proposal to translocate some of the lions from their
natural habitat in Gujarat to which one of the following sites?
(a) Corbett National Park
(b) Kuno Palpur Wildlife Sanctuary
(c) Mudumalai Wildlife Sanctuary
(d) Sariska National Park
Answer- b
Standard Explanation
An environment ministry’s
expert committee has approved
Kuno Palpur in Madhya Pradesh
as the second home for Asiatic
lions found only in Gir national
park.
IAS Ranker's Inputs
Read newspaper regularly and
make short notes of such
news. Further, mark such
places on map which will help
recalling it during the exam.
Let Civilsdaily help you with
this
K Siddhartha Sir's Inputs
The question framers set out to
complicate the question but ended up
simplifying.
Level : High
Q.4 The term ‘M-STrIPES’ is sometimes seen in the news in the context of
a) Captive breeding of Wild Fauna
b) Maintenance of Tiger Reserves
c) Indigenous Satellite Navigation System
d) Security of National Highways
Answer- b
Standard Explanation
The full form of M-STrIPES is
Monitoring System for Tigers’-
Intensive Protection and
Ecological Status. It’s a software
monitoring system launched by
the Indian Government in 2010
in some tiger reserves.
The aim is to reduce
vulnerability of Tigers.
IAS Ranker's Inputs
Here, Option c can be
eliminated by the fact that
NAVIC is the India's
navigation system.
K Siddhartha Sir's Inputs
Again this question signifies the
novelty of approach in asking
questions on Ecology, S&T and
Geography, and imagine the options
that are given to confuse the students
to the maximum possible extent.
Level :
Medium
Q.5 In the context of mitigating the impending global warming due to anthropogenic
emission of carbon dioxide, which of the following can be the potential sites for
carbon sequestration?
1. Abandoned and Uneconomic coal seams
2. Depleted oil and gas reservoirs
3. Subterranean deep saline formations
Select the correct answer using the code given below:
a) 1 and 2 only
b) 3 only
c) 1 and 3 only
d) 1, 2 and 3
Answer- d
Standard Explanation
Capturing CO2 from the major
stationary sources and its storage
into deep geological formations is
considered as a potential
mitigation option. Geological
storage of CO2 can be undertaken
in variety of geological settings in
sedimentary basins.
www.civilsdaily.com/lets-know-
about-co-sequestration/
IAS Ranker's Inputs
Carbon sequestration is one of
the important techniques
being proposed to correct
global warming. For prelims,
all those techniques must be
read. Other techniques include
Geoengineering, artificial tree,
Blue carbon project etc.
K Siddhartha Sir's Inputs
The question is not asking for
general suggestions, rather a specific
way to mitigate global warming.
More and more questions on
ecosystem management are going to
be in vogue. A solution oriented
thinking must for answering
questions. It is at next level beyond
intelligence.
Level :
Medium
Q.6 It is possible to produce algae based biofuels, but what is/are the likely
limitation(s) of developing countries in promoting this industry?
1. Production of algae based biofuels is possible in seas only and not on continents.
2. Setting up and engineering the algae based biofuel production requires high level of
expertise/technology until the construction is completed.
3. Economically viable production necessitates the setting up of large scale facilities
which may raise ecological and social concerns.
Select the correct answer using the code given below:
a) 1 and 2 only
b) 2 and 3 only
c) 3 only
d) 1, 2 and 3
Answer- b
Standard Explanation
Algae can be grown using land and
water not suitable for plant and
food production. The algal biofuel
technology includes selection of
specific species for production and
extraction of valuable co-products.
To make the industry
commercially viable, researchers
have pursued biotech varieties,
which could be particularly
dangerous if released into nature.
IAS Ranker's Inputs
For prelims, one must prepare
all 4 generations bio fuels. The
difference between Biofuel,
Biodiesel, Green Diesel must
be known
K Siddhartha Sir's Inputs
Complete solution oriented and
management oriented thinking.
Very deep, insightful and very
applied.
These questions are the future of
questions. Representative of what
will be the type of questions that
can be asked.
Level :
Medium
Q.7 Consider the following statements in respect of Trade Related Analysis of
Fauna and Flora in Commerce (TRAFFIC):
1. TRAFFIC is a bureau under United Nations Environment Programme (UNEP).
2. The Mission of TRAFFIC is to ensure that trade in wild plants and animals is not
a threat to the conservation of nature.
Which of the above statements is/are correct?
(a) 1 only
(b) 2 only
(c) Both 1 and 2
(d) Neither 1 nor 2
Answer- b
Standard Explanation
TRAFFIC, the wildlife trade
monitoring network, works to
ensure that trade in wild plants
and animals is not a threat to the
conservation of nature.
TRAFFIC also works in close co-
operation with the Secretariat of
the Convention on International
Trade in Endangered Species of
Wild Fauna and Flora (CITES).
IAS Ranker's Inputs
Make a gist of all important
organisation working for
environment protection,
sustainable development,
wildlife protection. For
prelims, know about their
headquarters, structure, their
area of work.
Every year, UPSC asks at least
one organisation from this
area.
K Siddhartha Sir's Inputs
Fantastic question, fact based,
observation based, interest based
and contemporary based.
Such questions are always going to
be difficult to answer.
Just see the explanation, which is
indicative of so many
amalgamations.
Part 5 - History
High and Medium level
Questions
History Sub Topics No. of questions
Ancient India 1
Art and Culture 5
Indian National Movement- 1915-1947 2
Modern India- 1857-1915 3
Pre Independence - Charters etc. 1
Sectors of Economy 1
Grand Total 13
Level : High
Q.1 Which of the following is/are famous for Sun temples?
1. Arasavalli
2. Amarakantak
3. Omkareshwar
Select the correct answer using the code given below :
(a) 1 only
(b) 2 and 3 only
(c) 1 and 3 only
(d) 1, 2 and 3
Answer- a
Standard Explanation
Amarkantak and Omkareshwar
do not have Sun Temples.
Also, they are situated on
Narmada coast and can be
related to Narmada Seva Yatra
held in M.P. , conclusion of
which was done by PM Modi.
IAS Ranker's Inputs
Note important temples
across India.
K Siddhartha Sir's Inputs
(EIBQU)
Level : High
Q.2 Consider the following pairs :
Traditions Communities
1. Chaliha Sahib Festival — Sindhis
2. Nanda Raj Jaat Yatra — Gonds
3. Wari-Warkari — Santhals
Which of the pairs given above is/are correctly matched?
(a) 1 only
(b) 2 and 3 only
(c) 1 and 3 only
(d) None of the above
Answer- a
Standard Explanation
Nanda Devi Raj Jaat Yatra is a
festival of Gharwali and kumaoni
people in Uttarakhand.
Wari Warkari or Varkari
(Pandharpur Wari) is a
Maharashtrian festival celebrated
by Marathas.
IAS Ranker's Inputs
Note important cultural
traditions of various
communities/states.
K Siddhartha Sir's Inputs
Deep diversified Knowledge
Strong base
Level : High
Q.3 With reference to the religious history of India, consider the following
statements :
1. Sautrantika and Samitiya were the sects of Jainism.
2. Sarvastivadin held that the constituents of phenomena were not wholly
momentary, but existed forever in a latent form.
Which of the statements given above is/are correct?
(a) 1 only
(b) 2 only
(c) Both 1 and 2
(d) Neither 1 nor 2
Answer- b
Standard Explanation
The two major sects include –
Digambar and Shwetambar.
95% of the practices followed are
same in both the sects.
IAS Ranker's Inputs
Buddhism and Jainism are
most important in UPSC.
K Siddhartha Sir's Inputs
Involvement with the
understanding
Interest & curiosity & Ability
to sustain
Level : High
Q.4 Which one of the following was a very important seaport in the Kakatiya
kingdom ?
(a) Kakinada
(b) Motupalli
(c) Machilipatnam (Masulipatnam)
(d) Nelluru
Answer- d
Standard Explanation
The Kakatiya dynasty was a
South Indian dynasty whose
capital was Orugallu, now known
as Warangal.
IAS Ranker's Inputs
Not solvable by any means.
Very factual.
K Siddhartha Sir's Inputs
Strong Information base
Level : High
Q.5 Consider the following statements:
1. The Factories Act, 1881 was passed with a view to fix the wages of industrial
workers and to allow the workers to form trade unions.
2. N.M. Lokhande was a pioneer in organizing the labour movement in British India.
Which of the above statements is/are correct?
a) 1 only
b) 2 only
c) Both 1 and 2
d) Neither 1 nor 2
Answer- b
Standard Explanation
Narayan Meghaji Lokhande
(1848-1897) was a pioneer of the
labour movement in India.
During Lord Ripon’s time, the
first Factories Act was adopted in
1881. The result of these
enactments was the limitation on
the factory working hours.
IAS Ranker's Inputs
Important acts of British and
important leaders of freedom
struggles need to be noted.
http://www.civilsdaily.com/blog
/list-of-important-indian-
freedom-fighters-and-their-
activities/
http://www.civilsdaily.com/blog
/list-of-acts-and-charters-of-the-
british-government-in-india/
K Siddhartha Sir's Inputs
A complete change in the type of
questions asked. Having known
the story of National Movement,
minute understanding and acts
must catch the attention because
these are the questions that the
answers of which reflect the ability
of picking a pattern among a
complex maze. This specific
quality is intrinsic to girls.
Level : High
Q.6 The Trade Disputes Act of 1929 provided for
a) the participation of workers in the management of industries.
b) arbitrary powers to the management to quell industrial disputes.
c) an intervention by the British Court in the event of a trade dispute.
d) a system of tribunals and a ban on strikes.
Answer- a
Standard Explanation
It provided for the participation
of workers in the management of
industries
IAS Ranker's Inputs
Important acts of British period
need to be noted.
http://www.civilsdaily.com/blog
/list-of-acts-and-charters-of-the-
british-government-in-india/
K Siddhartha Sir's Inputs
The way history is studied needs to
be changed.
The attitude required is now is
that how British rule affected us,
all our or made new laws, where
are its roots? Need to go the roots.
The question could not have been
answered only by a cosmetic study.
Level : Medium
Q.7 Who among the following was/were associated with the introduction of
Ryotwari Settlement in India during the British rule?
1. Lord Cornwallis
2. Alexander Read
3. Thomas Munro
Select the correct answer using the code given below:
(a) 1 only
(b) 1 and 3 only
(c) 2 and 3 only
(d) 1, 2 and 3
Answer- c
Standard Explanation
Lord Cornwallis is associated
with zamindari system.
IAS Ranker's Inputs
Read properly about
agricultural systems introduced
by British - Zamindari,
Ryotwari, Mahalwari.
One must know their region,
year of introduction and
implication.
K Siddhartha Sir's Inputs
Strong Information base
Recording Minute aspects
Level : Medium
Q.8 With reference to the difference between the culture of Rigvedic Aryans and Indus Valley
people, which of the following statements is/are correct?
1. Rigvedic Aryans used the coat of mail and helmet in warfare whereas the people of Indus
Valley Civilization did not leave any evidence of using them.
2. Rigvedic Aryans knew gold, silver and copper whereas Indus Valley people knew only
copper and iron.
3. Rigvedic Aryans had domesticated the horse whereas there is no evidence of Indus Valley
people having been aware of this animal.
Select the correct answer using the code given below:
(a) 1 only
(b) 2 and 3 only
(c) 1 and 3 only
(d) 1, 2 and 3
Answer- c
Standard Explanation
Indus people were basically peace
loving. The Aryans, on the contrary,
were warlike people and were
conversant with all kinds of
traditional arms and armour and
had devised a full-fledged ‘science of
war’.
The horse, which played a decisive
role in the Aryan system of warfare,
was not known to the Indus people.
IAS Ranker's Inputs
If one read options carefully,
few options can be eliminated.
For example, here statement 1
was difficult, but by eliminating
options (b) and (d), checking
correctness of Statement 1
becomes irrelevant.
K Siddhartha Sir's Inputs
The answer to this topic required
an intensive knowledge, and a very
close passionate observation about
both civilizations. This requires
taking pride in our country that is
India. It's now an essential
prerequisite.
There are going to be good
number of questions that can be
deciphered only when one is in
love with their country.
Level : Medium
Q.9 Consider the following pairs:
1. Radhakanta Deb – First President of the British Indian Association
2. Gazulu Lakshmi Narasu Chetty – Founder of the Madras Mahajana Sabha
3. Surendranath Banerjee – Founder of the Indian Association
Which of the above pairs is/are correctly matched?
(a) 1 only
(b) 1 and 3 only
(c) 2 and 3 only
(d) 1, 2 and 3
Answer- b
Standard Explanation
Madras Mahajana Sabha was
established by M.
Veeraraghavachary, G.
Subramania Iyer and P. Ananda
Charlu in May 1884.
IAS Ranker's Inputs
Make list of important
organisations during British
rule.
About these organisations,
know about their founders,
their area of work, their active
period, their substantial
contribution.
K Siddhartha Sir's Inputs
Fact and information based.
The Team
K Siddhartha, Mentor
He has over 20 years
experience in teaching Earth
Sciences, Life Sciences and
Behavioural traits in India’s
topmost institutes, and 15
years experience in training
students for Civil Services.
Praveen Verma, IAS
Achieved AIR 264 in CSE-
2016, AIR 22 in IFS-2016
B.Tech, IIT D | ex-
Powergrid
Sajal Singh
Economics (Hons.) from
DU | PG from Gokhale
institute of Economics and
Politics | Founding member
@http://www.electionpromis
estracker.in/
Tanay Rathi
Seasoned Civil Service
Aspirant
Chief Curator @Civilsdaily
Proposed deliverables
Deliverable 1
Lorem ipsum dolor sit amet
Sed do eiusmod tempor incididunt ut labore
Deliverable 2
Lorem ipsum dolor sit amet
Sed do eiusmod tempor incididunt ut labore
Deliverable 3
Lorem ipsum dolor sit amet
Sed do eiusmod tempor incididunt ut labore
Deliverable 4
Lorem ipsum dolor sit amet
Sed do eiusmod tempor incididunt ut labore

More Related Content

What's hot

Sandeep project toyota
Sandeep project toyotaSandeep project toyota
Sandeep project toyotaRajath Sandeep
 
Merchant Banking and Financial Services November 2020 MCQ type
Merchant Banking and Financial Services November 2020 MCQ typeMerchant Banking and Financial Services November 2020 MCQ type
Merchant Banking and Financial Services November 2020 MCQ typeGanesha Pandian
 
5 year analysis financial-managment-idlc-bank
5 year analysis financial-managment-idlc-bank 5 year analysis financial-managment-idlc-bank
5 year analysis financial-managment-idlc-bank istiuq ahmed
 
Keynote capitals india morning note march 26-'12
Keynote capitals india morning note march 26-'12Keynote capitals india morning note march 26-'12
Keynote capitals india morning note march 26-'12Keynote Capitals Ltd.
 
Expectations from Union Budget 2016
Expectations from Union Budget 2016Expectations from Union Budget 2016
Expectations from Union Budget 2016Abhirup Lahiri
 
Information brochure-economic-survey-&-budget
Information brochure-economic-survey-&-budgetInformation brochure-economic-survey-&-budget
Information brochure-economic-survey-&-budgetVVR IAS Exam Preparation
 
Life Insurance in India, Key Trends and Opportunities to 2016
Life Insurance in India, Key Trends and Opportunities to 2016Life Insurance in India, Key Trends and Opportunities to 2016
Life Insurance in India, Key Trends and Opportunities to 2016ReportsnReports
 
Doing Business in India
Doing Business in IndiaDoing Business in India
Doing Business in IndiaPuratan Bharti
 
WIPRO PROJECT FILE
WIPRO PROJECT FILEWIPRO PROJECT FILE
WIPRO PROJECT FILEHarsh Yadav
 
Aricles of FII
Aricles of FIIAricles of FII
Aricles of FIIvinod1505
 
Annual report2012 en
Annual report2012 enAnnual report2012 en
Annual report2012 enitechghana
 
Tax Litigation in India - EY India
Tax Litigation in India - EY IndiaTax Litigation in India - EY India
Tax Litigation in India - EY IndiaSadanandGahivare
 
Performance evaluation of_pnb
Performance evaluation of_pnbPerformance evaluation of_pnb
Performance evaluation of_pnbshruti_kapur
 

What's hot (19)

Sandeep project toyota
Sandeep project toyotaSandeep project toyota
Sandeep project toyota
 
Merchant Banking and Financial Services November 2020 MCQ type
Merchant Banking and Financial Services November 2020 MCQ typeMerchant Banking and Financial Services November 2020 MCQ type
Merchant Banking and Financial Services November 2020 MCQ type
 
5 year analysis financial-managment-idlc-bank
5 year analysis financial-managment-idlc-bank 5 year analysis financial-managment-idlc-bank
5 year analysis financial-managment-idlc-bank
 
Keynote capitals india morning note march 26-'12
Keynote capitals india morning note march 26-'12Keynote capitals india morning note march 26-'12
Keynote capitals india morning note march 26-'12
 
Economic Capsule - November 2012
Economic Capsule - November 2012Economic Capsule - November 2012
Economic Capsule - November 2012
 
Expectations from Union Budget 2016
Expectations from Union Budget 2016Expectations from Union Budget 2016
Expectations from Union Budget 2016
 
Information brochure-economic-survey-&-budget
Information brochure-economic-survey-&-budgetInformation brochure-economic-survey-&-budget
Information brochure-economic-survey-&-budget
 
Life Insurance in India, Key Trends and Opportunities to 2016
Life Insurance in India, Key Trends and Opportunities to 2016Life Insurance in India, Key Trends and Opportunities to 2016
Life Insurance in India, Key Trends and Opportunities to 2016
 
Financial management
Financial managementFinancial management
Financial management
 
Economic Capsule - February 2013
Economic Capsule - February 2013Economic Capsule - February 2013
Economic Capsule - February 2013
 
Doing Business in India
Doing Business in IndiaDoing Business in India
Doing Business in India
 
WIPRO PROJECT FILE
WIPRO PROJECT FILEWIPRO PROJECT FILE
WIPRO PROJECT FILE
 
CII Policy Watch July 2015
CII Policy Watch July 2015CII Policy Watch July 2015
CII Policy Watch July 2015
 
Aricles of FII
Aricles of FIIAricles of FII
Aricles of FII
 
FICCI UNEP | India Inquiry on Designing a Sustainable Financial System for India
FICCI UNEP | India Inquiry on Designing a Sustainable Financial System for IndiaFICCI UNEP | India Inquiry on Designing a Sustainable Financial System for India
FICCI UNEP | India Inquiry on Designing a Sustainable Financial System for India
 
Annual report2012 en
Annual report2012 enAnnual report2012 en
Annual report2012 en
 
Tax Litigation in India - EY India
Tax Litigation in India - EY IndiaTax Litigation in India - EY India
Tax Litigation in India - EY India
 
Performance evaluation of_pnb
Performance evaluation of_pnbPerformance evaluation of_pnb
Performance evaluation of_pnb
 
Npa of commercial banks
Npa of commercial banksNpa of commercial banks
Npa of commercial banks
 

Similar to IAS Prelims 2017 Analysis by Civilsdaily

Indian Economy 2022 ppt for Competitive Exams.pptx
Indian Economy 2022 ppt for Competitive Exams.pptxIndian Economy 2022 ppt for Competitive Exams.pptx
Indian Economy 2022 ppt for Competitive Exams.pptxLt Col (Dr) Satish Dhage
 
ICAI Elections -Your Vote will make the Difference
ICAI Elections -Your Vote will make the DifferenceICAI Elections -Your Vote will make the Difference
ICAI Elections -Your Vote will make the DifferenceNeha Sharma
 
IDFC-FIRST-Bank-Investor-Presentation-Q4-FY23.pdf
IDFC-FIRST-Bank-Investor-Presentation-Q4-FY23.pdfIDFC-FIRST-Bank-Investor-Presentation-Q4-FY23.pdf
IDFC-FIRST-Bank-Investor-Presentation-Q4-FY23.pdfBhushanKale32
 
Indian Construction Equipment and Infrastructure Financing Market
Indian Construction Equipment and Infrastructure Financing MarketIndian Construction Equipment and Infrastructure Financing Market
Indian Construction Equipment and Infrastructure Financing MarketNiraj Singhvi
 
Ma0044 institutional banking
Ma0044  institutional bankingMa0044  institutional banking
Ma0044 institutional bankingsmumbahelp
 
Fin4 dev develTHE RESTRUCTURING OFFICIAL DEVELOPMENT ASSISTANCE (ODA) CRITE...
Fin4 dev   develTHE RESTRUCTURING OFFICIAL DEVELOPMENT ASSISTANCE (ODA) CRITE...Fin4 dev   develTHE RESTRUCTURING OFFICIAL DEVELOPMENT ASSISTANCE (ODA) CRITE...
Fin4 dev develTHE RESTRUCTURING OFFICIAL DEVELOPMENT ASSISTANCE (ODA) CRITE...Vincent Moola MCIPS, Msc
 
Rbi in need to act efficiently, effectively and pragmatically
Rbi  in need to act efficiently, effectively and pragmaticallyRbi  in need to act efficiently, effectively and pragmatically
Rbi in need to act efficiently, effectively and pragmaticallyNeha Sharma
 
Rbi in need to act efficiently, effectively and pragmatically
Rbi  in need to act efficiently, effectively and pragmaticallyRbi  in need to act efficiently, effectively and pragmatically
Rbi in need to act efficiently, effectively and pragmaticallyAICAS
 
Case Study Example 1.pdf P a g e 1 Strat.docx
Case Study Example 1.pdf  P a g e   1 Strat.docxCase Study Example 1.pdf  P a g e   1 Strat.docx
Case Study Example 1.pdf P a g e 1 Strat.docxwendolynhalbert
 
CAMEL ANALYSIS OF NBFCS IN TAMILNADU
CAMEL ANALYSIS OF NBFCS IN TAMILNADUCAMEL ANALYSIS OF NBFCS IN TAMILNADU
CAMEL ANALYSIS OF NBFCS IN TAMILNADUIAEME Publication
 
CCXG Global Forum March 2018, Capacity-Building Needed and Received – Lebanon...
CCXG Global Forum March 2018, Capacity-Building Needed and Received – Lebanon...CCXG Global Forum March 2018, Capacity-Building Needed and Received – Lebanon...
CCXG Global Forum March 2018, Capacity-Building Needed and Received – Lebanon...OECD Environment
 
CCXG Global Forum March 2018, Capacity-Building Needed and Received – Lebanon...
CCXG Global Forum March 2018, Capacity-Building Needed and Received – Lebanon...CCXG Global Forum March 2018, Capacity-Building Needed and Received – Lebanon...
CCXG Global Forum March 2018, Capacity-Building Needed and Received – Lebanon...OECD Environment
 
state-of-india-fintech-union-2022.pdf
state-of-india-fintech-union-2022.pdfstate-of-india-fintech-union-2022.pdf
state-of-india-fintech-union-2022.pdfssuserdf252e
 
IDFC Core Equity Fund _Fund presentation
IDFC Core Equity Fund _Fund presentationIDFC Core Equity Fund _Fund presentation
IDFC Core Equity Fund _Fund presentationJubiIDFCEquity
 

Similar to IAS Prelims 2017 Analysis by Civilsdaily (20)

Indian Economy 2022 ppt for Competitive Exams.pptx
Indian Economy 2022 ppt for Competitive Exams.pptxIndian Economy 2022 ppt for Competitive Exams.pptx
Indian Economy 2022 ppt for Competitive Exams.pptx
 
ICAI Elections -Your Vote will make the Difference
ICAI Elections -Your Vote will make the DifferenceICAI Elections -Your Vote will make the Difference
ICAI Elections -Your Vote will make the Difference
 
IDFC-FIRST-Bank-Investor-Presentation-Q4-FY23.pdf
IDFC-FIRST-Bank-Investor-Presentation-Q4-FY23.pdfIDFC-FIRST-Bank-Investor-Presentation-Q4-FY23.pdf
IDFC-FIRST-Bank-Investor-Presentation-Q4-FY23.pdf
 
Business-studies-PQ.pdf
Business-studies-PQ.pdfBusiness-studies-PQ.pdf
Business-studies-PQ.pdf
 
Indian Construction Equipment and Infrastructure Financing Market
Indian Construction Equipment and Infrastructure Financing MarketIndian Construction Equipment and Infrastructure Financing Market
Indian Construction Equipment and Infrastructure Financing Market
 
Ease of Doing Business
Ease of Doing Business Ease of Doing Business
Ease of Doing Business
 
Ma0044 institutional banking
Ma0044  institutional bankingMa0044  institutional banking
Ma0044 institutional banking
 
Fin4 dev develTHE RESTRUCTURING OFFICIAL DEVELOPMENT ASSISTANCE (ODA) CRITE...
Fin4 dev   develTHE RESTRUCTURING OFFICIAL DEVELOPMENT ASSISTANCE (ODA) CRITE...Fin4 dev   develTHE RESTRUCTURING OFFICIAL DEVELOPMENT ASSISTANCE (ODA) CRITE...
Fin4 dev develTHE RESTRUCTURING OFFICIAL DEVELOPMENT ASSISTANCE (ODA) CRITE...
 
brsr.pptx
brsr.pptxbrsr.pptx
brsr.pptx
 
Rbi in need to act efficiently, effectively and pragmatically
Rbi  in need to act efficiently, effectively and pragmaticallyRbi  in need to act efficiently, effectively and pragmatically
Rbi in need to act efficiently, effectively and pragmatically
 
Rbi in need to act efficiently, effectively and pragmatically
Rbi  in need to act efficiently, effectively and pragmaticallyRbi  in need to act efficiently, effectively and pragmatically
Rbi in need to act efficiently, effectively and pragmatically
 
Multilateral Newsletter February 2016
Multilateral Newsletter February 2016Multilateral Newsletter February 2016
Multilateral Newsletter February 2016
 
Cobp Vie 2007
Cobp Vie 2007Cobp Vie 2007
Cobp Vie 2007
 
Case Study Example 1.pdf P a g e 1 Strat.docx
Case Study Example 1.pdf  P a g e   1 Strat.docxCase Study Example 1.pdf  P a g e   1 Strat.docx
Case Study Example 1.pdf P a g e 1 Strat.docx
 
CAMEL ANALYSIS OF NBFCS IN TAMILNADU
CAMEL ANALYSIS OF NBFCS IN TAMILNADUCAMEL ANALYSIS OF NBFCS IN TAMILNADU
CAMEL ANALYSIS OF NBFCS IN TAMILNADU
 
CCXG Global Forum March 2018, Capacity-Building Needed and Received – Lebanon...
CCXG Global Forum March 2018, Capacity-Building Needed and Received – Lebanon...CCXG Global Forum March 2018, Capacity-Building Needed and Received – Lebanon...
CCXG Global Forum March 2018, Capacity-Building Needed and Received – Lebanon...
 
CCXG Global Forum March 2018, Capacity-Building Needed and Received – Lebanon...
CCXG Global Forum March 2018, Capacity-Building Needed and Received – Lebanon...CCXG Global Forum March 2018, Capacity-Building Needed and Received – Lebanon...
CCXG Global Forum March 2018, Capacity-Building Needed and Received – Lebanon...
 
state-of-india-fintech-union-2022.pdf
state-of-india-fintech-union-2022.pdfstate-of-india-fintech-union-2022.pdf
state-of-india-fintech-union-2022.pdf
 
Policy Watch, September 2017
Policy Watch, September 2017Policy Watch, September 2017
Policy Watch, September 2017
 
IDFC Core Equity Fund _Fund presentation
IDFC Core Equity Fund _Fund presentationIDFC Core Equity Fund _Fund presentation
IDFC Core Equity Fund _Fund presentation
 

Recently uploaded

Field Attribute Index Feature in Odoo 17
Field Attribute Index Feature in Odoo 17Field Attribute Index Feature in Odoo 17
Field Attribute Index Feature in Odoo 17Celine George
 
call girls in Kamla Market (DELHI) 🔝 >༒9953330565🔝 genuine Escort Service 🔝✔️✔️
call girls in Kamla Market (DELHI) 🔝 >༒9953330565🔝 genuine Escort Service 🔝✔️✔️call girls in Kamla Market (DELHI) 🔝 >༒9953330565🔝 genuine Escort Service 🔝✔️✔️
call girls in Kamla Market (DELHI) 🔝 >༒9953330565🔝 genuine Escort Service 🔝✔️✔️9953056974 Low Rate Call Girls In Saket, Delhi NCR
 
Influencing policy (training slides from Fast Track Impact)
Influencing policy (training slides from Fast Track Impact)Influencing policy (training slides from Fast Track Impact)
Influencing policy (training slides from Fast Track Impact)Mark Reed
 
Proudly South Africa powerpoint Thorisha.pptx
Proudly South Africa powerpoint Thorisha.pptxProudly South Africa powerpoint Thorisha.pptx
Proudly South Africa powerpoint Thorisha.pptxthorishapillay1
 
Introduction to ArtificiaI Intelligence in Higher Education
Introduction to ArtificiaI Intelligence in Higher EducationIntroduction to ArtificiaI Intelligence in Higher Education
Introduction to ArtificiaI Intelligence in Higher Educationpboyjonauth
 
Earth Day Presentation wow hello nice great
Earth Day Presentation wow hello nice greatEarth Day Presentation wow hello nice great
Earth Day Presentation wow hello nice greatYousafMalik24
 
Computed Fields and api Depends in the Odoo 17
Computed Fields and api Depends in the Odoo 17Computed Fields and api Depends in the Odoo 17
Computed Fields and api Depends in the Odoo 17Celine George
 
How to Configure Email Server in Odoo 17
How to Configure Email Server in Odoo 17How to Configure Email Server in Odoo 17
How to Configure Email Server in Odoo 17Celine George
 
Hierarchy of management that covers different levels of management
Hierarchy of management that covers different levels of managementHierarchy of management that covers different levels of management
Hierarchy of management that covers different levels of managementmkooblal
 
Employee wellbeing at the workplace.pptx
Employee wellbeing at the workplace.pptxEmployee wellbeing at the workplace.pptx
Employee wellbeing at the workplace.pptxNirmalaLoungPoorunde1
 
What is Model Inheritance in Odoo 17 ERP
What is Model Inheritance in Odoo 17 ERPWhat is Model Inheritance in Odoo 17 ERP
What is Model Inheritance in Odoo 17 ERPCeline George
 
EPANDING THE CONTENT OF AN OUTLINE using notes.pptx
EPANDING THE CONTENT OF AN OUTLINE using notes.pptxEPANDING THE CONTENT OF AN OUTLINE using notes.pptx
EPANDING THE CONTENT OF AN OUTLINE using notes.pptxRaymartEstabillo3
 
Judging the Relevance and worth of ideas part 2.pptx
Judging the Relevance  and worth of ideas part 2.pptxJudging the Relevance  and worth of ideas part 2.pptx
Judging the Relevance and worth of ideas part 2.pptxSherlyMaeNeri
 
MULTIDISCIPLINRY NATURE OF THE ENVIRONMENTAL STUDIES.pptx
MULTIDISCIPLINRY NATURE OF THE ENVIRONMENTAL STUDIES.pptxMULTIDISCIPLINRY NATURE OF THE ENVIRONMENTAL STUDIES.pptx
MULTIDISCIPLINRY NATURE OF THE ENVIRONMENTAL STUDIES.pptxAnupkumar Sharma
 
Gas measurement O2,Co2,& ph) 04/2024.pptx
Gas measurement O2,Co2,& ph) 04/2024.pptxGas measurement O2,Co2,& ph) 04/2024.pptx
Gas measurement O2,Co2,& ph) 04/2024.pptxDr.Ibrahim Hassaan
 
ROOT CAUSE ANALYSIS PowerPoint Presentation
ROOT CAUSE ANALYSIS PowerPoint PresentationROOT CAUSE ANALYSIS PowerPoint Presentation
ROOT CAUSE ANALYSIS PowerPoint PresentationAadityaSharma884161
 

Recently uploaded (20)

Field Attribute Index Feature in Odoo 17
Field Attribute Index Feature in Odoo 17Field Attribute Index Feature in Odoo 17
Field Attribute Index Feature in Odoo 17
 
call girls in Kamla Market (DELHI) 🔝 >༒9953330565🔝 genuine Escort Service 🔝✔️✔️
call girls in Kamla Market (DELHI) 🔝 >༒9953330565🔝 genuine Escort Service 🔝✔️✔️call girls in Kamla Market (DELHI) 🔝 >༒9953330565🔝 genuine Escort Service 🔝✔️✔️
call girls in Kamla Market (DELHI) 🔝 >༒9953330565🔝 genuine Escort Service 🔝✔️✔️
 
Model Call Girl in Bikash Puri Delhi reach out to us at 🔝9953056974🔝
Model Call Girl in Bikash Puri  Delhi reach out to us at 🔝9953056974🔝Model Call Girl in Bikash Puri  Delhi reach out to us at 🔝9953056974🔝
Model Call Girl in Bikash Puri Delhi reach out to us at 🔝9953056974🔝
 
Model Call Girl in Tilak Nagar Delhi reach out to us at 🔝9953056974🔝
Model Call Girl in Tilak Nagar Delhi reach out to us at 🔝9953056974🔝Model Call Girl in Tilak Nagar Delhi reach out to us at 🔝9953056974🔝
Model Call Girl in Tilak Nagar Delhi reach out to us at 🔝9953056974🔝
 
Influencing policy (training slides from Fast Track Impact)
Influencing policy (training slides from Fast Track Impact)Influencing policy (training slides from Fast Track Impact)
Influencing policy (training slides from Fast Track Impact)
 
Proudly South Africa powerpoint Thorisha.pptx
Proudly South Africa powerpoint Thorisha.pptxProudly South Africa powerpoint Thorisha.pptx
Proudly South Africa powerpoint Thorisha.pptx
 
Introduction to ArtificiaI Intelligence in Higher Education
Introduction to ArtificiaI Intelligence in Higher EducationIntroduction to ArtificiaI Intelligence in Higher Education
Introduction to ArtificiaI Intelligence in Higher Education
 
Earth Day Presentation wow hello nice great
Earth Day Presentation wow hello nice greatEarth Day Presentation wow hello nice great
Earth Day Presentation wow hello nice great
 
Computed Fields and api Depends in the Odoo 17
Computed Fields and api Depends in the Odoo 17Computed Fields and api Depends in the Odoo 17
Computed Fields and api Depends in the Odoo 17
 
TataKelola dan KamSiber Kecerdasan Buatan v022.pdf
TataKelola dan KamSiber Kecerdasan Buatan v022.pdfTataKelola dan KamSiber Kecerdasan Buatan v022.pdf
TataKelola dan KamSiber Kecerdasan Buatan v022.pdf
 
How to Configure Email Server in Odoo 17
How to Configure Email Server in Odoo 17How to Configure Email Server in Odoo 17
How to Configure Email Server in Odoo 17
 
Raw materials used in Herbal Cosmetics.pptx
Raw materials used in Herbal Cosmetics.pptxRaw materials used in Herbal Cosmetics.pptx
Raw materials used in Herbal Cosmetics.pptx
 
Hierarchy of management that covers different levels of management
Hierarchy of management that covers different levels of managementHierarchy of management that covers different levels of management
Hierarchy of management that covers different levels of management
 
Employee wellbeing at the workplace.pptx
Employee wellbeing at the workplace.pptxEmployee wellbeing at the workplace.pptx
Employee wellbeing at the workplace.pptx
 
What is Model Inheritance in Odoo 17 ERP
What is Model Inheritance in Odoo 17 ERPWhat is Model Inheritance in Odoo 17 ERP
What is Model Inheritance in Odoo 17 ERP
 
EPANDING THE CONTENT OF AN OUTLINE using notes.pptx
EPANDING THE CONTENT OF AN OUTLINE using notes.pptxEPANDING THE CONTENT OF AN OUTLINE using notes.pptx
EPANDING THE CONTENT OF AN OUTLINE using notes.pptx
 
Judging the Relevance and worth of ideas part 2.pptx
Judging the Relevance  and worth of ideas part 2.pptxJudging the Relevance  and worth of ideas part 2.pptx
Judging the Relevance and worth of ideas part 2.pptx
 
MULTIDISCIPLINRY NATURE OF THE ENVIRONMENTAL STUDIES.pptx
MULTIDISCIPLINRY NATURE OF THE ENVIRONMENTAL STUDIES.pptxMULTIDISCIPLINRY NATURE OF THE ENVIRONMENTAL STUDIES.pptx
MULTIDISCIPLINRY NATURE OF THE ENVIRONMENTAL STUDIES.pptx
 
Gas measurement O2,Co2,& ph) 04/2024.pptx
Gas measurement O2,Co2,& ph) 04/2024.pptxGas measurement O2,Co2,& ph) 04/2024.pptx
Gas measurement O2,Co2,& ph) 04/2024.pptx
 
ROOT CAUSE ANALYSIS PowerPoint Presentation
ROOT CAUSE ANALYSIS PowerPoint PresentationROOT CAUSE ANALYSIS PowerPoint Presentation
ROOT CAUSE ANALYSIS PowerPoint Presentation
 

IAS Prelims 2017 Analysis by Civilsdaily

  • 1. IAS Prelims 2017 Analysis Civilsdaily.com
  • 2. Overview Civilsdaily has been trying to gauge trends followed by USPC in recent years by applying various Data interpretation tools. This is one such attempt to delve more into UPSC analysis and get some insights about various factors involved in Prelims papers.
  • 4. Subject 2011 2012 2013 2014 2015 2016 2017 History 11 19 16 20 17 15 13 Geography 11 17 18 14 16 7 10 Polity 12 20 16 14 13 7 26 Economy 19 17 19 10 13 18 8 E & B 15 17 17 18 11 18 10 Science 19 9 14 16 8 8 10 CA + GK 13 1 0 8 22 27 23
  • 5. How have divisions been done? The divisions are All subjects bifurcated according to their static and dynamic Subject wise analysis- Division into sub-subjects, difficulty levels, static and dynamic divisions and others.
  • 6. Trend analysis Findings Current Affairs have been increasing year on year and even static questions have effect of CA on them. Implications: More focus on CA has to made Comprehensive plan is needed where CA go along with static portion and not separately. Static Dynamic (CA based) 2014 92 8 2015 78 22 2016 73 27 2017 78 22
  • 11. Part 1 - Economy
  • 12. High and Medium level Questions
  • 13. Economics Sub Topics Count of Question Static/Dynamic Difficulty Level Banking, Finance 1 Static Medium Budget, Fiscal Policy 1 Static Medium Important Intl. Organizations 2 Static Medium Infrastructure 1 Static Medium Organizations involved 2 Static Medium Sectors of Economy 1 Static High Grand Total 8
  • 14. Level : High Q.1 Consider the following statements: 1. Tax revenue as a percent of GDP of India has steadily increased in the last decade. 2. Fiscal deficit as a percent of GDP of India has steadily increased in the last decade. Which of the statements given- above is/are correct ? (a) 1 only (b) 2 only (c) Both 1 and 2 (d) Neither 1 nor 2
  • 15. Answer- a Standard Explanation Fiscal Deficit has decreased in the last decade. www.civilsdaily.com/blog/ discussing-budget-2016-17- fiscal-discipline/ IAS Ranker's Inputs Read Economic survey and make short notes to tackle such questions. K Siddhartha Sir's Inputs Difficult if you haven’t followed the Eco Survey over time. Needs your ability to remember growth trends for atleast 2+ years
  • 16. Level : Medium Q.2 Consider the following statements: 1. The Standard Mark of Bureau of Indian Standards (BIS) is mandatory for automotive tyres and tubes. 2. AGMARK is a quality Certification Mark issued by the Food and Agriculture Organisation (FAO). Which of the statements given above is/are correct? (a) 1 only (b) 2only (c) Both 1 and 2 (d) Neither 1 nor 2
  • 17. Answer- a Standard Explanation Under Product Certification , Mandatory certification is there for Automobile Accessories such as Automotive vehicles – Tubes and pneumatic tyres AGMARK (Agricultural Mark) is a certification issued by Directorate of Marketing and Inspection for agricultural produce. IAS Ranker's Inputs There are some popular certifications such as AGMARK, GRIHA rating, gold's Hallmark etc. One should know basics about them. Also, BIS is important Indian agency. One should have idea about its functioning, its ministry etc. K Siddhartha Sir's Inputs AGMARK is by India which eliminates option (b) and (c).
  • 18. Level : Medium Q.3 With reference to the ‘National Intellectual Property Rights Policy’, consider the following statements: 1. It reiterates India’s commitment to the Doha Development Agenda and the TRIPS Agreement. 2. Department of Industrial Policy and Promotion is the nodal agency for regulating intellectual property rights in India. Which of the above statements is/are correct? (a) 1 only (b) 2 only (c) Both 1 and 2 (d) Neither 1 nor 2
  • 19. Answer- c Standard Explanation "The National IPR Policy recognizes that India has a well-established TRIPS- compliant legislative, administrative and judicial framework to safeguard IPRs. It reiterates India’s commitment to the Doha Development Agenda and the TRIPS agreement. It is monitored by DIPP (the nodal department to coordinate, guide and oversee implementation and future development of IPRs in India) www.civilsdaily.com/story/intellectual- property-rights-regime/ IAS Ranker's Inputs For prelims, one must know following points about important government policies - nodal agencies, targets, related global agreements/conventions . K Siddhartha Sir's Inputs DIPP monitors IPR issues (Remember in context of GI items etc.)
  • 20. Level : Medium Q.4 What is the purpose of setting up of Small Finance Banks (SFBs) in India? 1. To supply credit to small business units 2. To supply credit to small and marginal farmers 3. To encourage young entrepreneurs to set up business particularly in rural areas. Select the correct answer using the code given below: (a) 1 and 2 only (b) 2 and 3 only (c) 1 and 3 only (d) 1, 2 and 3
  • 21. Answer- a Standard Explanation "Local focus and ability to serve smaller customers will be a key criterion in licensing such banks. The bank shall primarily undertake basic banking activities of accepting deposits and lending to small farmers, small businesses, micro and small industries, and unorganized sector entities. www.civilsdaily.com/small- finance-banks-background/ IAS Ranker's Inputs For prelims, one must know recent schemes in details - their purpose, beneficiary, nodal agency, funds (centre : state) K Siddhartha Sir's Inputs In such options format, prove any one of the statements wrong. Here, statement 3 is wrong which leaves only option (a).
  • 22. Level : Medium Q.5 Which of the following statements is/are correct regarding the Monetary Policy Committee (MPC)? 1. It decides the RBI’s benchmark interest rates. 2. It is a 12-member body including the Governor of RBI and is reconstituted every year. 3. It functions under the chairmanship of the Union Finance Minister. Select the correct answer using the code given below: (a) 1 only (b) 1 and 2 only (c) 3 only (d) 2 and 3 only
  • 23. Answer- a Standard Explanation The Monetary Policy Committee would be entrusted with the task of fixing the benchmark policy rate (repo rate) required to contain inflation within the specified target level. It consists of 6 members. The Governor of the Bank— Chairperson, ex officio www.civilsdaily.com/centre- notifies-amended-rbi-act-mpc/ IAS Ranker's Inputs Whenever new institution is created by government, one must know the purpose of creation, its structure, and ministry under which it functions, mode of creation (statutory, executive, constitutional) etc. K Siddhartha Sir's Inputs (EIBQ).
  • 24. Level : Medium Q.6 Consider the following statements: 1. India has ratified the Trade Facilitation Agreement (TFA) of WTO. 2. TFA is a part of WTO’s Bali Ministerial Package of 2013. 3. TFA came into force in January 2016. Which of the statements given above is/are correct? a) 1 and 2 only b) 1 and 3 only c) 2 and 3 only d) 1, 2 and 3
  • 25. Answer- a Standard Explanation WTO members concluded negotiations at the 2013 Bali Ministerial Conference on the landmark Trade Facilitation Agreement (TFA), which entered into force on 22 February 2017 following its ratification by two- thirds of the WTO membership. www.civilsdaily.com/story/wto- and-india/ IAS Ranker's Inputs TFA is important global agreement. For prelims, one must know the timeline and India's efforts to implement the agreement. K Siddhartha Sir's Inputs WTO works on the principle of consensus. India has ratified the TFA in mid 2016. So, it cannot come into force in Jan 2016. By that logic, Statement 3 is wrong and that leaves only option a as correct answer.
  • 26. Level : Medium Q.7 With reference to ‘National Investment and Infrastructure Fund’, which of the following statements is/are correct? 1. It is an organ of NITI Aayog. 2. It has a corpus of Rs. 4,00,000 crore at present. Select the correct answer using the code given below : (a) 1 only (b) 2 only (c) Both 1 and 2 (d) Neither 1 nor 2
  • 27. Answer- d Standard Explanation NIIF has been structured as a fund of funds and set up as Category II Alternate Investment Fund (AIF) under the Securities and Exchange Board of India (SEBI) Regulations. Total corpus of the fund is Rs. 40000 Crore. www.civilsdaily.com/learn-about- niif/ IAS Ranker's Inputs Make notes of important initiatives, their purpose, implementing agency, fund allocation etc. K Siddhartha Sir's Inputs (EIBQ) Further, 4 lakh crore seems very high figure. So, option d can be guessed by that logic.
  • 28. Level : Medium Q.8 "The Global Infrastructure Facility is a/an (a) ASEAN initiative to upgrade infrastructure in Asia and financed by credit from the Asian Development Bank. (b) World Bank collaboration that facilitates the preparation and structuring of complex infrastructure Public-Private Partnerships (PPPs) to enable mobilization of private sector and institutional investor capital. (c) Collaboration among the major banks of the world working with the OECD and focused on expanding the set of infrastructure projects that have the potential to mobilize private investment. (d) UNCTAD funded initiative that seeks to finance and facilitate infrastructure development in the world."
  • 29. Answer- b Standard Explanation The World Bank (WB) has launched the GIF to specifically cater to the infrastructure needs of the emerging economies and developing countries. The GIF will channel money towards bankable infrastructure project in such countries. IAS Ranker's Inputs Make notes of important initiatives, their purpose, implementing agency, fund allocation etc. K Siddhartha Sir's Inputs Very close and overlapping choices makes it tough reading and choosing with an equal pinch for knowledge and information. Difficult to answer.
  • 30. Part 2 - Polity
  • 31. High and Medium level Questions
  • 32. Polity Sub Topics No. of questions Amendments to constitution 1 Elections- LS, RS, States 3 Functions of Judiciary 1 Functions of Legislature 5 Fundamental Rights/DPSP 7 Govt. Schemes 7 Important articles/schedules 2 Important treaties/ protocols 1 Misc. 1 Organizations involved 1 Panchayati Raj- Local governance 1 Powers of different authorities 2 Grand Total 32
  • 33. Level : High Q.1 Who among the following can join the National Pension System (NPS)? (a) Resident Indian citizens only (b) Persons of age from 21 to 55 only (c) All State Government employees joining the services after the date of notification by the respective State Governments (d) All Central Governments Employees including those of Armed Forces joining the services on or after 1st April, 2004"
  • 34. Answer- c Standard Explanation "The Central Government had introduced the National Pension System (NPS) with effect from January 1, 2004 (except for armed forces). Subsequently, various State Governments adopted this architecture and implemented NPS with effect from different dates. www.civilsdaily.com/learn-about- national-pension-system/ IAS Ranker's Inputs Create a list of important schemes where basic details like beneficiary, nodal ministry, targets etc. Use CD’s Prelims Listicles for quick revision: http://www.civilsdaily.com/top ic/pre-compilations-govt- schemes-2016-17-in-quick- reference-excel-sheet/ K Siddhartha Sir's Inputs Pension has been a problem. Candidate has been asked the question to know of he/she showed enough concerns. His concerns and sensitivity about the problems of the society. His empathy would determine his observation and answer to this topics.
  • 35. Level : High Q.2 With reference to `Quality Council of India (QCI)’, consider the following Statements: 1. QCI was set up jointly by the Government of India and the Indian Industry. 2. Chairman of QCI is appointed by the Prime Minister on the recommendations of the industry to the Government. Which of the above statements is/are correct? (a) 1 only (b) 2 only (c) Both 1 and 2
  • 36. Answer- c Standard Explanation Formed in 1997, the Quality Council of India was set-up by the CII, the FICCI, and the Associated Chambers of Commerce and Industry. The Chairman of QCI is appointed by the Prime Minister on recommendation of the industry to the government. IAS Ranker's Inputs Read 'About Us' webpage from official website and make their short notes for prelims. Also, carefully observe in newspaper when a new appointment is made to any government agency. The QCI was in news related to Swachh Bharat Mission. K Siddhartha Sir's Inputs Pure, blue bloodied fact based question to know whether the student keeps abreast of certain things……or not.
  • 37. Level : High Q.3 ‘Democracy’s superior virtue lies in the fact that it calls into activity (a) the intelligence and character ordinary men and women. (b) the methods for strengthening executive leadership (c) a superior individual with dynamism and vision. (d) a band of dedicated party workers.
  • 38. Answer- a Standard Explanation A democracy is superior, because it allows a right to every adult citizen, without any educational and wealth criterion. This allows him to use his own intelligence in choosing representatives. IAS Ranker's Inputs The question required clarity in fundamentals of polity and careful and patient reading of options. To make clarity in polity, read some good polity books like NCERT, Subhash Kashyap. K Siddhartha Sir's Inputs This is almost funny. It is an essay type topic. Requires a deep thinking to answer. Fortunately the choices are not close. So, you could have used Tikdams to answer Such topics have made their entry into PT questions this time.
  • 39. Level : High Q.4 In the context of India, which one of the following is the correct relationship between Rights and Duties? (a) Rights are correlative with Duties. (b) Rights are personal and hence independent of society and Duties. (c) Rights, not Duties, are important for the advancement of the personality of the citizen. (d) Duties, not Rights, are important for the stability of the State.
  • 40. Answer- a Standard Explanation Rights and duties are two phases of the same thing. Rights are considered to be essential for the expansion of human personality. Rights can be enjoyed only in the world of duties. For every right there is corresponding duty. IAS Ranker's Inputs Fundamentals of Polity must be understood. This requires in-depth understanding K Siddhartha Sir's Inputs Understand this language. Unless the candidates decipher the language, it will be even more difficult now onwards to answer such questions.
  • 41. Level : High Q.5 With reference to the ‘Prohibition of Benami Property Transactions Act, 1988 (PBPT Act)’, consider the following statements: 1. A property transaction is not treated as a benami transaction if the owner of the property is not aware of the transaction. 2. Properties held benami are liable for confiscation by the Government. 3. The Act provides for three authorities for investigations but does not provide for any appellate mechanism. Which of the statements given above is/are correct? a) 1 only b) 2 only c) 1 and 3 only
  • 42. Answer- b Standard Explanation "Benami Transactions (Prohibition) Act, 1988 prohibits certain types of financial transactions. The act defines a ‘benami’ transaction as any transaction in which property is transferred to one person for a consideration paid by another person. The act bans all benami transactions and gives the government the right to recover property held benami without paying any compensation. www.civilsdaily.com/new-benami-law- to-come-into-effect-from-november-1/ IAS Ranker's Inputs The Act was recently amended and was major instrument in government's battle against black money. Use CD’s Prelims Listicles for quick revision: http://www.civilsdaily.com/top ic/pre-compilations-govt- schemes-2016-17-in-quick- reference-excel-sheet/ K Siddhartha Sir's Inputs Good analytical frame of mind and learning required for processing this information. NO easy choices. No easy options.
  • 43. Level : Medium Q.6 With reference to the Parliament of India, consider the following statements: 1. A private member’s bill is a bill presented by a Member of Parliament who is not elected but only nominated by the President of India. 2. Recently, a private member’s bill has been passed in the Parliament of India for the first time in its history. Which of the statements given above is/are correct? (a) 1 only (b) 2 only (c) Both 1 and 2
  • 44. Answer- d Standard Explanation Members of parliament other than ministers are called private members and bills presented by them are known as private member’s bills. Any MP can introduce a bill in parliament. Only 14 private members’ bills passed since Independence. The Rights of Transgender Persons Bill, 2014, is the first private member’s bill to get the upper house’s approval in the past 45 years. www.civilsdaily.com/topic/transgender-bill- key-issues-and-analysis/ IAS Ranker's Inputs The private member's bill was in news recently when Rajya Sabha passed a bill for Transgender Community. That made the basics of Private bills important for prelims. K Siddhartha Sir's Inputs Simple, straight and Basics. Need to study.
  • 45. Q.7 ‘Recognition of Prior Learning Scheme’ is sometimes mentioned in the news with reference to (a) Certifying the skills acquired by construction workers through traditional channels. (b) Enrolling the persons in Universities for distance learning programmes. (c) Reserving some skilled jobs to rural and urban poor in some public sector undertakings. (d) Certifying the skills acquired by trainees under the National Skill Development
  • 46. Answer- a Standard Explanation "Recognition of Prior Learning Scheme- Individuals with prior learning experience or skills shall be assessed and certified under the Recognition of Prior Learning (RPL) component of the Pradhan Mantri Kaushal Vikas Yojana (PMKVY). RPL aims to align the competencies of the unregulated workforce of the country to the NSQF (National Skills Qualification Framework) IAS Ranker's Inputs Here option A is correct. It is not about certifying trainees but those who already possess skills (ie prior learning). In addition, the scheme is launched for construction sector. K Siddhartha Sir's Inputs Previously, these questions would have been asked in match the following type. This has changed. Meaning pure factual content has given way to some degree of inspection, introspection and retrospection.
  • 47. Level : Medium Q.8 Local self-government can be best explained as an exercise in a) Federalism b) Democratic decentralization c) Administrative delegation d) Direct democracy
  • 48. Answer- b Standard Explanation Democratic decentralization means decentralization of power. The main purpose of democratic decentralization is to bring fundamental changes in the traditional outlook about the power structure of the government. IAS Ranker's Inputs Read and understand basics of Panchayati Raj System. For prelims, the evolution of PRI, the provisions of 73rd and 74th amendments, PESA 1996 must be known to aspirant. In addition to that, administrative machinery in 5th schedule and 6th schedule area must also be known. K Siddhartha Sir's Inputs The question is the Basis of Polity but with close observation. (EIBQ)
  • 49. Level : Medium Q.9 Consider the following statements: With reference to the Constitution of India, the Directive Principles of State Policy constitute limitations upon 1. legislative function 2. executive function Which of the above statements is/are correct? a) 1 only b) 2 only c) Both 1 and 2 d) Neither 1 nor 2
  • 50. Answer- d Standard Explanation DPSP does not place any limitation on legislative and executive function, it is simply a guideline. IAS Ranker's Inputs One cannot go judiciary for violation of DPSP by legislature or executive if they are not provided by any law. K Siddhartha Sir's Inputs The question is limitations of Directive Policy-- that means studying the topic closely, minutely. No hard work but very minute work. No memorization but a study in a calm atmosphere to avoid cluttering of mind.
  • 51. Q.10 Which of the following are the objectives of ‘National Nutrition Mission’? 1. To create awareness relating to malnutrition among pregnant women and lactating mothers. 2. To reduce the incidence of anaemia among young children, adolescent girls and women. 3. To promote the consumption of millets, coarse cereals and unpolished rice. 4. To promote the consumption of poultry eggs. Select the correct answer using the code given below: a) 1 and 2 only b) 1, 2 and 3 only c) 1, 2 and 4 only
  • 52. Answer- a Standard Explanation The key objectives of this programme are : To create awareness relating to malnutrition amongst pregnant women, lactating mothers, promote healthy lactating practices and importance of balanced nutrition; To reduce incidence of anaemia among young children, adolescent girls and women. www.civilsdaily.com/op-ed-snap-need- nutrition-mission/ IAS Ranker's Inputs Make ministry wise short notes on schemes and missions. Read their objectives, beneficiaries, targets and implementing agencies. Make use of the last minute Prelims listicles K Siddhartha Sir's Inputs Straight simple, typical, representative question from Govt. Schemes.
  • 53. Level : Medium Q.11 Consider the following statements: 1. In the election for Lok Sabha or State Assembly, the winning candidate must get at least 50 percent of the votes polled, to be declared elected. 2. According to the provisions laid down in the Constitution of India, in Lok Sabha, the Speaker’s post goes to the majority party and the Deputy Speaker’s to the Opposition. Which of the statements given above is/are correct? a) 1 only b) 2 only c) Both 1 and 2 d) Neither 1 nor 2
  • 54. Answer- d Standard Explanation India follows First Past the Post system, there is no provision for 50% votes. There is no provision related to it in the Constitution. IAS Ranker's Inputs For prelims, one must be aware of the some Parliamentary practices whether they are based on convention, law or constitutional provision. Other examples include Zero hour, Deputy Chairman of Rajya Sabha etc. K Siddhartha Sir's Inputs In the first option, Provision of 50% post is essentially a ploy to confuse the candidates. The purpose of the question is to make sure whether the students are 100% sure about the answers or not.
  • 55. Level : Medium Q.12 What is the purpose of Vidyanjali Yojana’? 1. To enable the famous foreign campuses in India. 2. To increase the quality of education provided in government schools by taking help from the private sector and the community. 3. To encourage voluntary monetary contributions from private individuals and organizations so as to improve the infrastructure facilities for primary and secondary schools. Select the correct answer using the code given below: a) 2 only b) 3 only c) 1 and 2 only d) 2 and 3 only
  • 56. Answer- a Standard Explanation Vidyanjali is an initiative to enhance community and private sector involvement in Government run elementary schools across the country under the overall aegis of the Sarva Shiksha Abhiyan. www.civilsdaily.com/app-aid- volunteer-workers-govt- schools/ IAS Ranker's Inputs For prelims, one must know recent schemes in details - their purpose, beneficiary, nodal agency, funds (centre : state) K Siddhartha Sir's Inputs The question is Ok but the options are very close, very interrelated, very meaningful. Such genre of questions were very few last year but are more in number this year. You have to be thorough with your reading an revision.
  • 57. Level : Medium Q.13 Out of the following statements, choose the one that brings out the principle underlying the Cabinet form of Government : (a) An arrangement for minimizing the criticism against the Government whose responsibilities are complex and hard to carry out to the satisfaction of all. (b) A mechanism for speeding up the activities of the Government whose responsibilities are increasing day by day. (c) A mechanism of parliamentary democracy for ensuring collective responsibility of the Government to the people. (d) A device for strengthening the hands of the head of the Government whose hold over the people is in a state of decline.
  • 58. Answer- c Standard Explanation Council of Ministers is collectively responsible to Parliament (Lok Sabha) and Lok Sabha is elected by people themselves. IAS Ranker's Inputs Fundamental question from Polity. K Siddhartha Sir's Inputs Although this question was simple, Simple to ask, simple to answer and simple to think there were overtones to make it complicated.
  • 59. Level : Medium Q.14 With reference to ‘National Skills Qualification Framework (NSQF)’, which of the statements given below is/are correct? 1. Under NSQF, a learner can acquire the certification for competency only through formal learning. 2. An outcome expected from the implementation of NSQF is the mobility between vocational and general education. Select the correct answer using the code given below: (a) 1 only (b) 2 only (c) Both 1 and 2 (d) Neither 1 nor 2
  • 60. Answer- b Standard Explanation "The National Skills Qualifications Framework (NSQF) is a competency-based framework that organizes all qualifications according to a series of levels of knowledge, skills and aptitude. These levels, are defined in terms of learning outcomes which the learner must possess regardless of whether they are obtained through formal, non- formal or informal learning. IAS Ranker's Inputs Prepare notes on skill development in India. For prelims, it should include the current status of skills in India, current programmes run by government to improve skills like Skill India, the agencies involved in skill development in India. K Siddhartha Sir's Inputs Contemporary, Current Compact with deep information levels and differentiation skill, i.e. ability to differentiate between different type of choices.
  • 61. Level : Medium Q.15 Consider the following in respect of ‘National Career Service’ : 1. National Career Service is an initiative of the Department of Personnel and Training, Government of India. 2. National Career Service has been launched in a Mission Mode to improve the employment opportunities to uneducated youth of the country. Which of the above statements is/are correct ? (a) 1 only (b) 2 only (c) Both 1 and 2 (d) Neither 1 nor 2
  • 62. Answer- b Standard Explanation The Government is implementing the National Career Service Project for linking employment exchanges and other institutions using technology to provide a variety of employment related services like job postings, career counselling, vocational guidance, skill courses, apprenticeship, etc. www.civilsdaily.com/pib- implementation-of-national- career-service/ IAS Ranker's Inputs Important scheme related to employment. K Siddhartha Sir's Inputs The candidate need Concerns of unemployment, which is frequently covered in Newspaper as it also concerns the youth.
  • 63. Part 3 - Geography
  • 64. High and Medium level Questions
  • 65. Geography Sub Topics No. of Questions Agriculture- Irrigation, fertilizers etc. 1 Indian Geography-Forests 1 Indian Geography-Places 3 Indian Geography-Rivers 1 World Geography 4 Grand Total 10
  • 66. Level : High Q.1 From the ecological point of view, which one of the following assumes importance in being a good link between the Eastern Ghats and the Western Ghats? (a) Sathyamangalam Tiger Reserve (b) Nallamala Forest (c) Nagarhole National Park (d) Seshachalam Biosphere Reserve
  • 67. Answer- b Standard Explanation Sathyamangalam forest range is a significant wildlife corridor in the Nilgiri Biosphere Reserve between the Western Ghats and the rest of the Eastern Ghats. It is located in Tamil Nadu. http://www.civilsdaily.com/topic/ 15-nov-2016-prelims-daily-with- previous-year-questions-tikdams/ IAS Ranker's Inputs Mark important protected areas on blank maps. Along with that, know about important species in those areas. Revise them regularly. K Siddhartha Sir's Inputs Answers to these type of questions now requires identifying the most significant part, most unique part, i.e., with respect to its location, river, animals, flora, or any new development. For example asking something like northern most confluence town in India?
  • 68. Level : High Q.2 At one of the places in India, if you stand on the seashore and watch the sea, you will find that the sea water recedes from the shore line a few kilometers and comes back to the shore, twice a day, and you can actually walk on the seafloor when the water recedes. This unique phenomenon is seen at: a) Bhavnagar b) Bheemunipatnam c) Chandipur d) Nagapattinam
  • 69. Answer- c Standard Explanation The beach is unique in that the water recedes up to 5 kilometers during the ebb tide. When the tide is in, locals trawl for small fish along the coast. IAS Ranker's Inputs Very factual question. K Siddhartha Sir's Inputs No theory, no memorisation, no fact, but one unique information separated this beach out of context. This beach like Dhanu road is remarkable for a lot of things. Identification of uniqueness is one key factor that is emerging now.
  • 70. Level : High Q.3 Consider the following statements: 1. In India, the Himalayas are spread over five States only. 2. Western Ghats are spread over five States only. 3. Pulicat Lake is spread over two States only. Which of the statements given above is/are correct ? (a) 1 and 2 only (b) 3 only (c) 2 and 3 only (d) 1 and 3 only
  • 71. Answer- b Standard Explanation Western Ghats – The Ghats traverse the States of Kerala, Tamil Nadu, Karnataka, Goa, Maharashtra and Gujarat. Pulicat Lake is the second largest brackish water lake or lagoon in India, after Chilka Lake. It straddles the border of Andhra Pradesh and Tamil Nadu states. IAS Ranker's Inputs Factual question/map needs to be fully memorized in order to answer. K Siddhartha Sir's Inputs Observation based question, location based question, one that cannot be memorised.
  • 72. Level : Medium Q.4 Mediterranean Sea is a border of which of the following countries ? 1. Jordan 2. Iraq 3. Lebanon 4. Syria Select the correct answer using the code given below: (a) 1, 2 and 3 only (b) 2 and 3 only (c) 3 and 4 only (d) 1, 3 and 4 only
  • 73. Answer- c Standard Explanation The countries with coastlines on the Mediterranean Sea are Albania, Algeria, Bosnia and Herzegovina, Croatia, Cyprus, Egypt, France, Greece, Israel, Italy, Lebanon, Libya, Malta, Morocco, Monaco, Montenegro, Slovenia, Spain, Syria, Tunisia and Turkey. IAS Ranker's Inputs For prelims, carefully study the middle east, south east areas. K Siddhartha Sir's Inputs (EIBQ)
  • 74. Level : Medium Q.5 With reference to ‘Indian Ocean Dipole (IOD)’ sometimes mentioned in the news while forecasting Indian monsoon, which of the following statements is/are correct? 1. IOD phenomenon is characterized by a difference in sea surface temperature between tropical Western Indian Ocean and tropical Eastern Pacific Ocean. 2. An IOD phenomenon can influence an El Nino’s impact on the monsoon. Select the correct answer using the code given below: (a) 1 only (b) 2 only (c) Both 1 and 2 (d) Neither 1 nor 2
  • 75. Answer- b Standard Explanation The phenomenon is concerned to temperature differentials in Western Indian Ocean and Eastern Indian Ocean. www.civilsdaily.com/story/geog raphic-phenomenon-key- concepts-and-issues/ IAS Ranker's Inputs The term was in news last year for its impact on monsoon. Make notes of such important terms appearing in news. K Siddhartha Sir's Inputs Imagine the question, It’s not on El Nino, Not On ENSO, Not on Madden Julien oscillation, not on Aido Arabian Sea temperature inversion. Of all the factors that may have affected the monsoons the pic was dipole. Of course all them are linked by ENSO event.
  • 76. Level : Medium Q.6 Which of the following practices can help in water conservation in agriculture ? 1. Reduced or zero tillage of the land 2. Applying gypsum before irrigating the field 3. Allowing crop residue to remain in the field Select the correct answer using the code given below : (a) 1 and 2 only (b) 3 only (c) 1 and 3 only (d) 1, 2 and 3
  • 77. Answer- d Standard Explanation All these methods help in reducing water usage.. IAS Ranker's Inputs Agriculture consumes a major portion of water and due to scarcity of fresh water, its conservation techniques in agriculture become very important. For prelims, one must know about Climate smart agriculture, organic farming, sustainable agriculture etc. K Siddhartha Sir's Inputs A lot of discussion is carried on about saving water and the correct method of agriculture. So the students must understand the importance of water conservation particularly in agriculture where a lot of water is wasted.
  • 78. Part 4 - Environment & Biodiversity
  • 79. High and Medium level Questions
  • 80. E&B Sub Topics No. of questions Agriculture- Irrigation, fertilizers etc. 1 Basic terms related to E&B 1 Important articles/schedules 2 Important Intl. Organizations 1 Important treaties/ protocols 2 Indian Geography-Places 2 New Tech/ Innovation in news 1 Organizations involved 1 Static theory on E&B 1 Grand Total 12
  • 81. Level : High Q.1 Consider the following statements : 1. Climate and Clean Air Coalition (CCAC) to Reduce Short Lived Climate Pollutants is a unique initiative of G20 group of countries; 2. The CCAC focuses on methane, black carbon and hydrofluorocarbons. Which of the statements given above is/are correct ? (a) 1 only (b) 2 only (c) Both 1 and 2 (d) Neither 1 nor 2
  • 82. Answer- b Standard Explanation The Climate and Clean Air Coalition to Reduce Short-Lived Climate Pollutants (CCAC) was launched by the United Nations Environment Programme (UNEP) and six countries— Bangladesh, Canada, Ghana, Mexico, Sweden, and the United States—on 16 February 2012. It focuses on O3, CH4, Black Carbon, and HFCs. IAS Ranker's Inputs Since it is 4 - 5 years old news and India is not party to this coalition, It becomes very difficult to know about such question. In prelims, one must also know which questions to leave because there are negative marks too. One should not make total blind guesses. K Siddhartha Sir's Inputs The explanation says all. Of course, All ecological conventions taking place must be considered important. The back ground history for conventions need to be added. No more cosmetic preparation required.
  • 83. Level : High Q.1 With, reference to ‘Global Climate Change Alliance’ which of the following statements is/are correct? 1. It is an initiative of the European Union. 2. It provides technical and financial support to targeted developing countries to integrate climate change into their development policies and budgets. 3. It is coordinated by World Resources Institute (WRI) and World Business Council for Sustainable Development (WBCSD). Select the correct answer using the code given below : (a) 1 and 2 only (b) 3 only (c) 2 and 3 only (d) 1, 2 and 3
  • 84. Answer- a Standard Explanation "The Global Climate Change Alliance (GCCA) is an initiative of the European Union. Its overall objective is to build a new alliance on climate change between the European Union and the poor developing countries that are most affected and that have the least capacity to deal with climate change. IAS Ranker's Inputs Try getting answer from analysing options. K Siddhartha Sir's Inputs This question will make the student think for sometime before they can gather their nerve to answer it correctly, what a way to tease the students by putting option 3 and its associated institutions. Expect more from UPSC next year
  • 85. Level : High Q.3 Recently there was a proposal to translocate some of the lions from their natural habitat in Gujarat to which one of the following sites? (a) Corbett National Park (b) Kuno Palpur Wildlife Sanctuary (c) Mudumalai Wildlife Sanctuary (d) Sariska National Park
  • 86. Answer- b Standard Explanation An environment ministry’s expert committee has approved Kuno Palpur in Madhya Pradesh as the second home for Asiatic lions found only in Gir national park. IAS Ranker's Inputs Read newspaper regularly and make short notes of such news. Further, mark such places on map which will help recalling it during the exam. Let Civilsdaily help you with this K Siddhartha Sir's Inputs The question framers set out to complicate the question but ended up simplifying.
  • 87. Level : High Q.4 The term ‘M-STrIPES’ is sometimes seen in the news in the context of a) Captive breeding of Wild Fauna b) Maintenance of Tiger Reserves c) Indigenous Satellite Navigation System d) Security of National Highways
  • 88. Answer- b Standard Explanation The full form of M-STrIPES is Monitoring System for Tigers’- Intensive Protection and Ecological Status. It’s a software monitoring system launched by the Indian Government in 2010 in some tiger reserves. The aim is to reduce vulnerability of Tigers. IAS Ranker's Inputs Here, Option c can be eliminated by the fact that NAVIC is the India's navigation system. K Siddhartha Sir's Inputs Again this question signifies the novelty of approach in asking questions on Ecology, S&T and Geography, and imagine the options that are given to confuse the students to the maximum possible extent.
  • 89. Level : Medium Q.5 In the context of mitigating the impending global warming due to anthropogenic emission of carbon dioxide, which of the following can be the potential sites for carbon sequestration? 1. Abandoned and Uneconomic coal seams 2. Depleted oil and gas reservoirs 3. Subterranean deep saline formations Select the correct answer using the code given below: a) 1 and 2 only b) 3 only c) 1 and 3 only d) 1, 2 and 3
  • 90. Answer- d Standard Explanation Capturing CO2 from the major stationary sources and its storage into deep geological formations is considered as a potential mitigation option. Geological storage of CO2 can be undertaken in variety of geological settings in sedimentary basins. www.civilsdaily.com/lets-know- about-co-sequestration/ IAS Ranker's Inputs Carbon sequestration is one of the important techniques being proposed to correct global warming. For prelims, all those techniques must be read. Other techniques include Geoengineering, artificial tree, Blue carbon project etc. K Siddhartha Sir's Inputs The question is not asking for general suggestions, rather a specific way to mitigate global warming. More and more questions on ecosystem management are going to be in vogue. A solution oriented thinking must for answering questions. It is at next level beyond intelligence.
  • 91. Level : Medium Q.6 It is possible to produce algae based biofuels, but what is/are the likely limitation(s) of developing countries in promoting this industry? 1. Production of algae based biofuels is possible in seas only and not on continents. 2. Setting up and engineering the algae based biofuel production requires high level of expertise/technology until the construction is completed. 3. Economically viable production necessitates the setting up of large scale facilities which may raise ecological and social concerns. Select the correct answer using the code given below: a) 1 and 2 only b) 2 and 3 only c) 3 only d) 1, 2 and 3
  • 92. Answer- b Standard Explanation Algae can be grown using land and water not suitable for plant and food production. The algal biofuel technology includes selection of specific species for production and extraction of valuable co-products. To make the industry commercially viable, researchers have pursued biotech varieties, which could be particularly dangerous if released into nature. IAS Ranker's Inputs For prelims, one must prepare all 4 generations bio fuels. The difference between Biofuel, Biodiesel, Green Diesel must be known K Siddhartha Sir's Inputs Complete solution oriented and management oriented thinking. Very deep, insightful and very applied. These questions are the future of questions. Representative of what will be the type of questions that can be asked.
  • 93. Level : Medium Q.7 Consider the following statements in respect of Trade Related Analysis of Fauna and Flora in Commerce (TRAFFIC): 1. TRAFFIC is a bureau under United Nations Environment Programme (UNEP). 2. The Mission of TRAFFIC is to ensure that trade in wild plants and animals is not a threat to the conservation of nature. Which of the above statements is/are correct? (a) 1 only (b) 2 only (c) Both 1 and 2 (d) Neither 1 nor 2
  • 94. Answer- b Standard Explanation TRAFFIC, the wildlife trade monitoring network, works to ensure that trade in wild plants and animals is not a threat to the conservation of nature. TRAFFIC also works in close co- operation with the Secretariat of the Convention on International Trade in Endangered Species of Wild Fauna and Flora (CITES). IAS Ranker's Inputs Make a gist of all important organisation working for environment protection, sustainable development, wildlife protection. For prelims, know about their headquarters, structure, their area of work. Every year, UPSC asks at least one organisation from this area. K Siddhartha Sir's Inputs Fantastic question, fact based, observation based, interest based and contemporary based. Such questions are always going to be difficult to answer. Just see the explanation, which is indicative of so many amalgamations.
  • 95. Part 5 - History
  • 96. High and Medium level Questions
  • 97. History Sub Topics No. of questions Ancient India 1 Art and Culture 5 Indian National Movement- 1915-1947 2 Modern India- 1857-1915 3 Pre Independence - Charters etc. 1 Sectors of Economy 1 Grand Total 13
  • 98. Level : High Q.1 Which of the following is/are famous for Sun temples? 1. Arasavalli 2. Amarakantak 3. Omkareshwar Select the correct answer using the code given below : (a) 1 only (b) 2 and 3 only (c) 1 and 3 only (d) 1, 2 and 3
  • 99. Answer- a Standard Explanation Amarkantak and Omkareshwar do not have Sun Temples. Also, they are situated on Narmada coast and can be related to Narmada Seva Yatra held in M.P. , conclusion of which was done by PM Modi. IAS Ranker's Inputs Note important temples across India. K Siddhartha Sir's Inputs (EIBQU)
  • 100. Level : High Q.2 Consider the following pairs : Traditions Communities 1. Chaliha Sahib Festival — Sindhis 2. Nanda Raj Jaat Yatra — Gonds 3. Wari-Warkari — Santhals Which of the pairs given above is/are correctly matched? (a) 1 only (b) 2 and 3 only (c) 1 and 3 only (d) None of the above
  • 101. Answer- a Standard Explanation Nanda Devi Raj Jaat Yatra is a festival of Gharwali and kumaoni people in Uttarakhand. Wari Warkari or Varkari (Pandharpur Wari) is a Maharashtrian festival celebrated by Marathas. IAS Ranker's Inputs Note important cultural traditions of various communities/states. K Siddhartha Sir's Inputs Deep diversified Knowledge Strong base
  • 102. Level : High Q.3 With reference to the religious history of India, consider the following statements : 1. Sautrantika and Samitiya were the sects of Jainism. 2. Sarvastivadin held that the constituents of phenomena were not wholly momentary, but existed forever in a latent form. Which of the statements given above is/are correct? (a) 1 only (b) 2 only (c) Both 1 and 2 (d) Neither 1 nor 2
  • 103. Answer- b Standard Explanation The two major sects include – Digambar and Shwetambar. 95% of the practices followed are same in both the sects. IAS Ranker's Inputs Buddhism and Jainism are most important in UPSC. K Siddhartha Sir's Inputs Involvement with the understanding Interest & curiosity & Ability to sustain
  • 104. Level : High Q.4 Which one of the following was a very important seaport in the Kakatiya kingdom ? (a) Kakinada (b) Motupalli (c) Machilipatnam (Masulipatnam) (d) Nelluru
  • 105. Answer- d Standard Explanation The Kakatiya dynasty was a South Indian dynasty whose capital was Orugallu, now known as Warangal. IAS Ranker's Inputs Not solvable by any means. Very factual. K Siddhartha Sir's Inputs Strong Information base
  • 106. Level : High Q.5 Consider the following statements: 1. The Factories Act, 1881 was passed with a view to fix the wages of industrial workers and to allow the workers to form trade unions. 2. N.M. Lokhande was a pioneer in organizing the labour movement in British India. Which of the above statements is/are correct? a) 1 only b) 2 only c) Both 1 and 2 d) Neither 1 nor 2
  • 107. Answer- b Standard Explanation Narayan Meghaji Lokhande (1848-1897) was a pioneer of the labour movement in India. During Lord Ripon’s time, the first Factories Act was adopted in 1881. The result of these enactments was the limitation on the factory working hours. IAS Ranker's Inputs Important acts of British and important leaders of freedom struggles need to be noted. http://www.civilsdaily.com/blog /list-of-important-indian- freedom-fighters-and-their- activities/ http://www.civilsdaily.com/blog /list-of-acts-and-charters-of-the- british-government-in-india/ K Siddhartha Sir's Inputs A complete change in the type of questions asked. Having known the story of National Movement, minute understanding and acts must catch the attention because these are the questions that the answers of which reflect the ability of picking a pattern among a complex maze. This specific quality is intrinsic to girls.
  • 108. Level : High Q.6 The Trade Disputes Act of 1929 provided for a) the participation of workers in the management of industries. b) arbitrary powers to the management to quell industrial disputes. c) an intervention by the British Court in the event of a trade dispute. d) a system of tribunals and a ban on strikes.
  • 109. Answer- a Standard Explanation It provided for the participation of workers in the management of industries IAS Ranker's Inputs Important acts of British period need to be noted. http://www.civilsdaily.com/blog /list-of-acts-and-charters-of-the- british-government-in-india/ K Siddhartha Sir's Inputs The way history is studied needs to be changed. The attitude required is now is that how British rule affected us, all our or made new laws, where are its roots? Need to go the roots. The question could not have been answered only by a cosmetic study.
  • 110. Level : Medium Q.7 Who among the following was/were associated with the introduction of Ryotwari Settlement in India during the British rule? 1. Lord Cornwallis 2. Alexander Read 3. Thomas Munro Select the correct answer using the code given below: (a) 1 only (b) 1 and 3 only (c) 2 and 3 only (d) 1, 2 and 3
  • 111. Answer- c Standard Explanation Lord Cornwallis is associated with zamindari system. IAS Ranker's Inputs Read properly about agricultural systems introduced by British - Zamindari, Ryotwari, Mahalwari. One must know their region, year of introduction and implication. K Siddhartha Sir's Inputs Strong Information base Recording Minute aspects
  • 112. Level : Medium Q.8 With reference to the difference between the culture of Rigvedic Aryans and Indus Valley people, which of the following statements is/are correct? 1. Rigvedic Aryans used the coat of mail and helmet in warfare whereas the people of Indus Valley Civilization did not leave any evidence of using them. 2. Rigvedic Aryans knew gold, silver and copper whereas Indus Valley people knew only copper and iron. 3. Rigvedic Aryans had domesticated the horse whereas there is no evidence of Indus Valley people having been aware of this animal. Select the correct answer using the code given below: (a) 1 only (b) 2 and 3 only (c) 1 and 3 only (d) 1, 2 and 3
  • 113. Answer- c Standard Explanation Indus people were basically peace loving. The Aryans, on the contrary, were warlike people and were conversant with all kinds of traditional arms and armour and had devised a full-fledged ‘science of war’. The horse, which played a decisive role in the Aryan system of warfare, was not known to the Indus people. IAS Ranker's Inputs If one read options carefully, few options can be eliminated. For example, here statement 1 was difficult, but by eliminating options (b) and (d), checking correctness of Statement 1 becomes irrelevant. K Siddhartha Sir's Inputs The answer to this topic required an intensive knowledge, and a very close passionate observation about both civilizations. This requires taking pride in our country that is India. It's now an essential prerequisite. There are going to be good number of questions that can be deciphered only when one is in love with their country.
  • 114. Level : Medium Q.9 Consider the following pairs: 1. Radhakanta Deb – First President of the British Indian Association 2. Gazulu Lakshmi Narasu Chetty – Founder of the Madras Mahajana Sabha 3. Surendranath Banerjee – Founder of the Indian Association Which of the above pairs is/are correctly matched? (a) 1 only (b) 1 and 3 only (c) 2 and 3 only (d) 1, 2 and 3
  • 115. Answer- b Standard Explanation Madras Mahajana Sabha was established by M. Veeraraghavachary, G. Subramania Iyer and P. Ananda Charlu in May 1884. IAS Ranker's Inputs Make list of important organisations during British rule. About these organisations, know about their founders, their area of work, their active period, their substantial contribution. K Siddhartha Sir's Inputs Fact and information based.
  • 116. The Team K Siddhartha, Mentor He has over 20 years experience in teaching Earth Sciences, Life Sciences and Behavioural traits in India’s topmost institutes, and 15 years experience in training students for Civil Services. Praveen Verma, IAS Achieved AIR 264 in CSE- 2016, AIR 22 in IFS-2016 B.Tech, IIT D | ex- Powergrid Sajal Singh Economics (Hons.) from DU | PG from Gokhale institute of Economics and Politics | Founding member @http://www.electionpromis estracker.in/ Tanay Rathi Seasoned Civil Service Aspirant Chief Curator @Civilsdaily
  • 117. Proposed deliverables Deliverable 1 Lorem ipsum dolor sit amet Sed do eiusmod tempor incididunt ut labore Deliverable 2 Lorem ipsum dolor sit amet Sed do eiusmod tempor incididunt ut labore Deliverable 3 Lorem ipsum dolor sit amet Sed do eiusmod tempor incididunt ut labore Deliverable 4 Lorem ipsum dolor sit amet Sed do eiusmod tempor incididunt ut labore